You are on page 1of 70

OPHTHALMOLOGY

KMU PAST PAPERS SOLVED SEQS

M. SHAH FAHAD | REHMAN MEDICAL COLLEGE


No part of this publication is to be reproduced
for purposes of sale, you may transmit by any
form or by any means, electronic, mechanical,
photocopying, recording or otherwise, without
the prior permission of the author, this eBook is
specifically for educational purposes only.
Contents 1

EYE LIDS AND LACRIMAL SYSTEM 2

CONJUNCTIVA, CORNEA AND SCLERA 6

GLAUCOMA 21

UVEA, LENS AND VITREOUS 30

RETINA AND OPTIC NERVE 40

ORBIT AND INJURY TO EYE 52

OCULAR MOBILITY AND STRABISMUS 63

References
1. Kanski Clinical Ophthalmology 8th Edition
2. Clinical Ophthalmology 5th Edition by Shafi M. Jatoi
3. Jogi Basic Ophthalmology 4th Edition

KMU Past Papers Solved SEQs


Eye lids and Lacrimal system 2

Q-1. 50 year Old woman complaints of watering and discharge from her left eye for last 6
months. Irrigation of the lacrimal sac produce mucopurulent reflex.
a) Give causes of epiphora.
b) What is the most likely diagnosis?
c) What is the treatment?

ANS:
a) CAUSES OF EPIPHORA:
1. Congenital Absence of Lacrimal Puncta
2. Congenital Non-canalization or delayed canalization of Nasolacrimal Duct
3. Occlusion of Puncta – Foreign Body, hair, etc
4. Chronic Dacryocystitis with Nasolacrimal Duct Blockage
5. Growth or Inflammation of Inferior Meatus
6. Functional insufficiency for draining the tear in to the nose
b) CHRONIC DACRYOCYCTITIS
c) TREATMENT:
1. Repeating syringing of the nasolacrimal duct and frequent instillation of antibiotic drops
(in recent cases)
2. In recalcitrant Cases:
i) Dacryocystectomy
ii) Dacryocystorhinostomy
iii) Insertion of Special tubes (Lester-Jones tube, Pawar’s Cannula in Lacrimal Sac)

Q-2. A 65-year-Old man complained of a foreign body sensation in his right eye and tearing-the
right eye showed early cataracts with lower lid turned in and eye lashes rubbing On globe.
a) What is your diagnosis?
b) What are different types of the condition?
c) What is your differential diagnosis?
d) How will you treat the patient?

ANS:
a) ENTROPION
b) DIFFERENT TYPES OF THE CONDITION:
1. Involutional entropion (Senile type or age related)
2. Cicatricical entropion which is caused by scarring of palpebral conjunctiva
3. Spastic
4. Mechanical type
5. Congenital type
c) DIFFERENTIAL DIAGNOSIS:
1. Trichiasis
2. Pseudotrichiasis
3. Districhiasis
d) TREATMENT OPTIONS:
 Non-surgical:
1. Lubricants,
2. Taping,
3. Soft bandage
4. Contact lenses

KMU Past Papers Solved SEQs


Eye lids and Lacrimal system 3

5. Orbicularis chemo-denervation with botulinum toxin injection.


 Surgical:
1. Transverse everting sutures prevent over-riding of the preseptal orbicularis.
2. The Wies Procedure (It consists of full-thickness horizontal lid-splitting and insertion of
everting sutures)
3. Lower lid retractor reinsertion
4. Lateral canthal sling and full-thickness lateral pentagon excision to correct Horizontal lid
laxity
5. The Fox Procedure
6. Modified Wheelers Operation
7. For cicatricial entropion (mild cases) is by transverse tarsotomy with anterior rotation of
the lid margin, (severe cases) replacing deficient or keratinized conjunctiva and replacing
the scarred and contracted tarsal plate with composite grafts.

Q-3. A 65-year elderly female presented with tearing and pain in the right eye for last four
months. Her vision is 6/6 both eyes and purulent material was expressed by applying
pressure over right lacrimal sac area
a) What is your diagnosis?
b) What are the causes of this condition?
c) How you treat the patient?

ANS:
a) CHRONIC DACRYOCYSTITIS
b) CAUSES:
1. Foreign Body which leads to obstruction
2. Chronic or Recurrent Unilateral Conjunctivitis
3. Congenital and Acquired Obstruction of Nasolacrimal Duct
4. Preseptal Cellulitis
5. Bacterial Infections i.e. Staphylococcal or Streptococcal
6. Growth in Lacrimal Sac
c) TREATMENT:
Medical:
1. Topical and Systemic Antibiotic
2. Improve lid hygiene
Surgical
1. Dacryocystorhinostomy for creating a passage between the lacrimal sac and nasal cavity
through bony ostium
2. External DCR when performed through skin approach
3. Transnasal laser DCR is performed through nasal approach this procedure avoid incision to
skin and scar
Treat underlying causes
1. Sinus Diseases
2. Growth in Nose or Lacrimal Sac
3. Chronic Conjunctivitis

Q-4. A 50 year old man has a chalazion removed from his upper eyelid three times, it has
recurred. How will you manage?

ANS:

KMU Past Papers Solved SEQs


Eye lids and Lacrimal system 4

This is sebaceous gland carcinoma (nodulare type)


Reason Why this is not chalizon:
Ist, chalazioin usually does not reoccur, second, chalizon is common on lower eye lid, third the age of
patient is 50 so chances of malignancy is more.
Management:
1. Biopsy which can be
 Incisional, using a blade in which only part of the lesion is removed for histological
diagnosis,
 Excisional, in which the entire lesion is removed
2. Surgical excision
3. Reconstruction
4. Exenteration
5. Radiotherapy

Q-5. A 40 years old banker developed foreign body sensation and tearing in her left eye. Ocular
examination revealed retraction of the right upper eye lid. Past history revealed treatment for
Grave's disease for one year.
a) List ocular features of this disease?
b) What treatment would you suggest for this patient?

ANS:
a) OCULAR FEATURES:
1. Eye-lid:
i) Retraction of Upper Eyelid (Dalrymple’s sign)
ii) Lid Lag (von-graefe’s sign)
iii) Infrequent Blinking (Stellwag’s sign)
iv) Weakness of Convergence (mobius sign)
v) Increased pigmentation of lids. (Jellinek’s sign)
i) Poor forehead wrinkling on looking up.( Joffroy’s sign)
2. Exophthalmos Bilateral or may be present unilateral with associated exposure keratitis.
3. Puffy and full eyelids
4. Conjunctival Congestion and Chemosis (edema)
5. Kerato-conjunctivitis sicca secondary to infiltration of lacrimal gland
6. Optic nerve dysfunction (papilloedema, papillitis, optic neuropathy)
7. Restrictive ocular mobility, diplopia due to raised intra-orbital pressure
b) TREATMENT:
 Medical
1. Control Hyperthyroidism:
i) Systemic iodide and anti-thyroid drugs are given in mild type.
ii) Thyroidectomy
iii) Radioactive iodine
2. Control ocular discomfort and dryness
3. Artificial tears and lubricants
4. Systemic corticosteroids help to reduce the oedema and infiltration.
5. Orbital radiation
 Surgery
1. Orbital decompression
2. Strabismus surgery
3. Lid-lengthening surgery

KMU Past Papers Solved SEQs


Eye lids and Lacrimal system 5

4. Blepharoplasty

Q-6. A 60 years old male patient presenting with right sided facial palsy.
a) What are the ocular complications?
b) How will you manage?

ANS:
a) OCULAR COMPLICATIONS:
1. Epiphora (Watering) caused by malposition of the inferior lacrimal punctum
2. Dryness of eye
3. Exposure keratopathy due to lagophthalmos
4. Corneal Ulcer
b) MANAGEMENT:
 History and examination:
 Investigation:
1. Investigate for the cause of Paralysis,
2. Imaging Studies MRI and CT scan for intracranial lesions
3. Topodiagnostic tests for localization of lesion
 Treatment:
Temporary: to protect the cornea in anticipation of spontaneous recovery of facial nerve
function.
1. Lubrication with higher viscosity tear substitutes during the day, with instillation of
ointment and taping shut of the lids during sleep,
2. Botulinum toxin injection into the levator to induce temporary ptosis.
3. Temporary tarsorrhaphy may be necessary, particularly in patients with a poor Bell
phenomenon.
Permanent: when there is irreversible damage to the facial nerve or when no further
improvement has occurred for 6-12 months in a Bell palsy.
1. Medial canthoplasty may be performed if the medial canthal tendon is intact.
2. A lateral canthal sling or tarsal strip may be used to correct residual ectropion and
raise the lateral canthus
3. Upper eyelid lowering by levator disinsertion.
4. Gold weight implantation in the upper lid can assist closure.
5. A small lateral tarsorrhaphy is usually cosmetically acceptable.

KMU Past Papers Solved SEQs


Conjunctiva, Cornea and Sclera 6

Q-1. A 65 years old farmer presents with fleshy conjunctival growth on nasal side of the cornea
encroaching upon limbus in his right eye.
a) What is most probable Diagnosis?
b) How vision can be affected?
c) How it is treated?

ANS:
a) PTERYGIUM
b) VISION IS AFFECTED AS:
1. Lesions may interfere with vision by obscuring the visual axis
2. Astigmatism due to traction of fibrovascular tissue on cornea.
Other complication:
1. Most small lesions are asymptomatic.
2. Irritation and grittiness are caused by a dellen – localized drying – effect at the advancing
edge due to interference with the pre-corneal tear film (more likely if the head of the
pterygium is especially elevated).
3. Patients who wear contact lenses may develop symptoms of irritation at an earlier stage
due to edge lift.
4. There may be intermittent inflammation similar to pingueculitis.
5. Cosmesis may be a significant problem.
6. Extensive lesions, particularly if recurrent, may be associated with subconjunctival
fibrosis extending to the fornices that may cause restricted ocular excursion.
7. If pseudo-pterygium is suspected, there may be a history of a causative episode.
c) TREATMENT:
Medical Treatment:
1. Tear substitute to avoid dryness
2. Topical steroids
3. The patient may be advised to wear sunglasses to reduce ultraviolet exposure in order to
decrease the growth stimulus
Surgical Treatment:
1. Simple excision (‘bare sclera’ technique) is associated with a high rate of recurrence
(around 80%), often with more aggressive behavior than the original lesion.
2. Simple conjunctival flap.
3. Conjunctival auto grafting, the donor conjunctival patch is usually harvested from the
superior or super temporal para-limbal the site generally heals rapidly, even without
suturing.
4. Amniotic membrane patch grafting is an alternative. Both conjunctival grafts and amniotic
membranes can be secured with tissue glue rather than sutured, shortening operating
time and reducing postoperative irritation.
5. Adjunctive treatment with mitomycin C
6. Beta irradiation are sometimes used in place of patching techniques.
7. Peripheral lamellar keratoplasty may be required for deep lesions
8. Argon laser to destroy regrowth of new vessels at limbus.

Q-2. A child age 1, month presented to you with hazy cornea


a) What are the differential diagnosis?
b) What important signs will you look for in the examination?

ANS:

KMU Past Papers Solved SEQs


Conjunctiva, Cornea and Sclera 7

a) DIFFERENTIAL DIAGNOSIS:
1. Scleral or Corneal trauma
2. Congenital glaucoma
3. Birth trauma
4. Ulcer (Keratitis)
5. Metabolic disease
6. Peter anomaly
7. Endothelial dystrophies
b) IMPORTANT SIGNS IN EXAMINATION:
1. Congested Disc with Small Hemorrhages
2. Spontaneous Pulsation of Central Retinal Artery

Q-3. A 9 years old child presents every summer with intense itching, lacrimation, photophobia
and foreign body sensation. On examination there is papillary hypertrophy on superior tarsus,
keratopathy of both eyes and plaque on cornea of left eye.
a) What is your diagnosis
b) How you will treat the patient?

ANS:
a) VERNAL KERATO-CONJUNCTIVITIS
a) TREATMENT:
 General measures
1. Allergen avoidance, if possible.
2. Cool compresses may be helpful.
3. Lid hygiene should be used for associated staphylococcal blepharitis.
4. Moisturizing cream can be applied to dry, fissured skin.
5. Bandage contact lens wear to aid healing of persistent epithelial defects.
 Local treatment
1. Mast cell stabilizers (sodium cromoglicate) reduce the frequency of acute exacerbations
and the need for steroids.
2. Topical antihistamines (epinastine) when used in isolation are about as effective as
mast cell stabilizers.
3. Combined antihistamine and vasoconstrictor (antazoline with xylometazoline) may
offer relief in some cases.
4. Combined action antihistamine/mast cell stabilizers (azelastine, olopatadine) are
helpful in many patients and have a relatively rapid onset of action.
5. Non-steroidal anti-inflammatory preparations (e.g. diclofenac) may improve comfort
by blocking non-histamine mediators.
6. Topical steroids (prednisolone) are used for
i) Severe exacerbations of conjunctivitis
ii) Significant keratopathy
7. Antibiotics may be used in conjunction with steroids in severe keratopathy to prevent or
treat bacterial infection.
8. Acetylcysteine is a mucolytic agent that is useful in VKC for dissolving mucus filaments
and deposits, and addressing early plaque formation.
9. Immune modulators Cyclosporine may be indicated if steroids are ineffective,
inadequate or poorly tolerated, or as a steroid-sparing agent in patients with severe
disease.
10. Supratarsal steroid injection may be considered in severe palpebral disease or for non-
compliant patients.
 Systemic treatment

KMU Past Papers Solved SEQs


Conjunctiva, Cornea and Sclera 8

1. Oral antihistamines help itching, promote sleep and reduce nocturnal eye rubbing.
2. Immunosuppressive agents (e.g. cyclosporine, azathioprine) Monoclonal antibodies
against T cells have shown some promise in refractory cases.
3. Other treatments that may be effective in some patients include aspirin in VKC, allergen
desensitization, and plasmapheresis in patients with high serum IgE levels.
 Surgical treatment
1. Debridement of large mucous plaques or shield ulcers.
2. Superficial keratectomy may be required to remove plaques and allow epithelialization.
Excimer laser phototherapeutic keratectomy is an alternative.
3. Surface maintenance/restoration surgery such as amniotic membrane overlay grafting
or lamellar keratoplasty, or eyelid procedures such as botulinum toxin-induced ptosis or
lateral tarsorrhaphy, may be required for severe persistent epithelial defects or
ulceration. Gluing may be appropriate for focal (‘punched-out’) corneal perforation.

Q-4. A child present with itching and redness of both his eyes since the start of early summer
this has happened in the past two summers as well.
a) What is the likely diagnosis?
b) Name the three conjunctival signs
c) Name 6 corneal complications

ANS:
a) VERNAL KERATO-CONJUNCTIVITS
b) CONJUNCTIVAL SIGNS:
1. Conjunctival Hyperaemia and Chemosis
2. Palperbral Conjunctiva shows multiple polygonal – shaped raised areas like cobblestones
(due to diffuse papillary hypertrophy)
3. Diffuse chalky white superficial spots (Horner – Tranta’s dots) may be seen on limbus
c) CORNEAL COMPLICATIONS:
1. Superficial Punctate Keratitis
2. Epithelial Macroerosion
3. Epithelial Ulceration (shield ulcers)
4. Plaque (caused by macroerosion of epithelium, it becomes coated with mucus)
5. Ring Scar (Result of Subepithelial Corneal Scaring)
6. Pseduogeron toxon (Cupid’s bow appearance)

KMU Past Papers Solved SEQs


Conjunctiva, Cornea and Sclera 9

Q-5. A boy 5 years of age develops sudden onset of redness, itching, burning sensation, stringy
discharge in both for last two days while visiting his grandparents in Islamabad. He also has a
history of asthma. He had the same eye symptoms last April which settled down after a few
days.
His older brother aged 15 years suffers from photophobia, burning and itching the entire year
with exacerbations in spring and summers. In addition, his brother's vision is progressively
deteriorating with frequent need for change of spectacles due to severe corneal astigmatism
and now necessitating contact lenses. He and his parents have been told that there is
possibility that the progressive corneal damage may require penetrating keratoplasty.
a) What is the probable diagnosis in the boy aged 5 years?
b) What is the probable diagnosis responsible for corneal astigmatism in his older brother?
c) What class of drugs are primarily used for treatment of 5 year old boy?
d) List 2 potential side effects of chronic use of topical steroids?
e) What are the different forms of Vernal kerato-conjunctivitis?

ANS:
a) VERNAL KERATO-CONJUNCTIVITS
b) KERATOCONUS
c) ANTIHISTAMINES
d) SIDE EFFECTS OF CHRONIC USE OF TOPICAL STEROIDS:
1. Steroid induced Glaucoma
2. Cataract
3. Temporary cloudy vision
4. Increased sensitivity to light (photophobia)
5. Foreign body sensation
6. Allergic reactions
e) FORMS OF VERNAL KERATO-CONJUNCTIVITIS:
1. Palpebral Form
2. Limbal Form
3. Mixed Form

Q-6. A farmer felt something went into his left into his left eye while cutting some bushes. The
following day he noticed irritation in the same eye. In 48 hours his eye started to get red and
his wife noticed a white spot on his left cornea. He travels to the local Ophthalmologist and the
following observations are made in the left eye. The Right eye is Visual Acuity: 6/9, lop
digitally normal, pupil: NO RAPD Slit Lamp: 1. Minimal Conjunctival injection 2. White
Feathery infiltrate of cornea 2mm wide in the infero-temporal quadrant of cornea
a) What is the Most Likely Diagnosis?
b) Enumerate 3 Differential Diagnosis?
c) The Ophthalmologist takes a corneal scrape and sends it to the Microbiologist to do some
tests on the specimen. Enumerate those test?
d) Enumerate Structures of the eye that can be secondarily affected by Keratitis?
e) Enumerate surgical treatment of Suppurative Corneal Perforation?

ANS:
a) FUNGAL KERATITIS
b) DIFFERENTIAL DIAGNOSIS:

KMU Past Papers Solved SEQs


Conjunctiva, Cornea and Sclera 10

1. Corneal ulcer,
2. Acute iridocyctitis,
3. Corneal foreign body,
4. Conjunctivitis
c) TESTS ON SPECIMEN:
1. Staining with Methamine Silver, Gram and Gimesa Stains
2. Culture in Sabouraud’s Medium, blood – agar plate or brain – heart infusion broth
d) STRUCTURES AFFECTED:
1. Cornea
2. Cilliary Muscles
3. Iris
4. Lids
e) SURGICAL TREATMENT FOR SUPPURATIVE CORNEAL PERFORATION:
1. Conjunctival flap—The non-healing ulcer may be covered with the conjunctival flap.
2. Therapeutic keratoplasty—Full thickness graft is applied to enhance healing and to prevent
perforation.
3. Tarsorrhaphy—It is done in cases of neuroparalytic ulcers and exposure keratitis

Q-7. An 8 years old boy is brought by his mother for itching and watering eyes for the last 3
months. The patient suffered same problem in the last spring also. On examination upper
tarsus shows giant papillae and mucoid discharge.
a) What is your diagnosis?
b) What is your differential diagnosis?
c) What is the treatment?

ANS:
a) VERNAL KERATO-CONJUCTIVITIS
b) DIFFERENTIAL DIAGNOSIS:
1. Atopic Kerato-conjunctivitis
2. Acute or sub-acute catarrhal conjunctivitis
3. Trachoma
4. Hay Fever Conjunctivitis
c) TREATMENT:
 General measures
6. Allergen avoidance, if possible.
7. Cool compresses may be helpful.
8. Lid hygiene should be used for associated staphylococcal blepharitis.
9. Moisturizing cream can be applied to dry, fissured skin.
10. Bandage contact lens wear to aid healing of persistent epithelial defects.
 Local treatment
1. Mast cell stabilizers (sodium cromoglicate) reduce the frequency of acute
exacerbations and the need for steroids.
2. Topical antihistamines (epinastine) when used in isolation are about as effective as
mast cell stabilizers.
3. Combined antihistamine and vasoconstrictor (antazoline with xylometazoline)
may offer relief in some cases.
4. Combined action antihistamine/mast cell stabilizers (azelastine, olopatadine) are
helpful in many patients and have a relatively rapid onset of action.

KMU Past Papers Solved SEQs


Conjunctiva, Cornea and Sclera 11

5. Non-steroidal anti-inflammatory preparations (e.g. diclofenac) may improve


comfort by blocking non-histamine mediators.
6. Topical steroids (prednisolone) are used for
iii) Severe exacerbations of conjunctivitis
iv) Significant keratopathy
7. Antibiotics may be used in conjunction with steroids in severe keratopathy to
prevent or treat bacterial infection.
8. Acetylcysteine is a mucolytic agent that is useful in VKC for dissolving mucus
filaments and deposits, and addressing early plaque formation.
9. Immune modulators Cyclosporine may be indicated if steroids are ineffective,
inadequate or poorly tolerated, or as a steroid-sparing agent in patients with severe
disease.
10. Supratarsal steroid injection may be considered in severe palpebral disease or for
non-compliant patients.
 Systemic treatment
1. Oral antihistamines help itching, promote sleep and reduce nocturnal eye rubbing.
2. Immunosuppressive agents (e.g. cyclosporine, azathioprine) Monoclonal antibodies
against T cells have shown some promise in refractory cases.
3. Other treatments that may be effective in some patients include aspirin in VKC
(avoided in children and adolescents due to Reye syndrome risk), allergen
desensitization, and plasmapheresis in patients with high serum IgE levels.
 Surgical treatment
1. Debridement of large mucous plaques or shield ulcers.
2. Superficial keratectomy may be required to remove plaques and allow
epithelialization. Excimer laser phototherapeutic keratectomy is an alternative.
3. Surface maintenance/restoration surgery such as amniotic membrane overlay
grafting or lamellar keratoplasty, or eyelid procedures such as botulinum toxin-
induced ptosis or lateral tarsorrhaphy, may be required for severe persistent
epithelial defects or ulceration. Gluing may be appropriate for focal (‘punched-out’)
corneal perforation.

Q-8. A life guard states that his eyes has been red for a long time. He has a wing shaped fold of
fibro-vascular tissue nasally in both eyes that extend onto the cornea.
a) What is the diagnosis
b) What are the etiological factors?
c) What are the preventive measures and treatment?

ANS
a) PTERYGIUM
b) ETIOLOGICAL FACTORS:
1. Hot climates which lead to drying of interpalpebral tear film
2. Ultraviolet light which damage corneal epithelium bowman’s membrane and corneal
stroma
3. Chronic infection of conjuncitva
4. Dusty environment
c) PREVENTIVE MEASURES AND TREATMENT:
1. Tear Substitute
2. Topical steroids
3. Sunglasses to reduce ultraviolet exposure
Treatment:
 Medical: as above

KMU Past Papers Solved SEQs


Conjunctiva, Cornea and Sclera 12

 Surgical:
1. Simple excision (‘bare sclera’ technique) is associated with a high rate of recurrence
(around 80%), often with more aggressive behavior than the original lesion
2. Simple conjunctival flap
3. Conjunctival auto grafting, the donor conjunctival patch is usually harvested from the
superior or super temporal para-limbal the site generally heals rapidly, even witho]ut
suturing
4. Amniotic membrane patch grafting is an alternative. Both conjunctival grafts and
amniotic membranes can be secured with tissue glue rather than sutured, shortening
operating time and reducing postoperative irritation
5. Adjunctive treatment with mitomycin C
6. Beta irradiation are sometimes used in place of patching techniques
7. Peripheral lamellar keratoplasty may be required for deep lesions
8. Argon laser to destroy regrowth of new vessels at limbus

Q-9. An 80 years old woman complains of red eyes that constantly tear and bum she also feels
foreign body sensation and reports that her vision is not clear as before. Vision varies with
tear blink. She has noticed condition over past several years
a) What is the diagnosis?
b) What are the three signs?
c) Name two tests relevant for diagnosis
d) What is the treatment?

ANS:
a) KERATOCONJUNCTIVITIS SICCA (SJOGREN’S SYNDROME)
b) SIGNS:
1. Bitot’s Spot (Small triangular, shiny, silver white patches seen on bulbar conjunctiva near
the outer canthus usually)
2. Conjuctival Epithelium becomes epidermoid like that of skin
3. Excessive mucus secretion (white colored) due to deficiency of aqueous layer
c) DIAGNOSIS:
1. Staining with Rose Bengal dye 1%
2. Tear film break – up time (BUT)
d) TREATMENT:
 Medical:
1. Tear Substitutes (Eyedrops, Eye Ointments, Insert)
2. Tear Preservation (Lower Lacrimal Punctum is blocked)
3. Treat basic cause e.g. Vitamin A deficiency, Trachoma etc
4. Dark glasses or eye shields should be worn
5. Contact Lenses
 Surgical Measures:
1. Mucous Membrane Grafting
2. Conjunctival Transplant
3. Amniotic Membrane Transplant Keratoprosthesis
4. Correction of Ectropion, entropion, trichiasis

KMU Past Papers Solved SEQs


Conjunctiva, Cornea and Sclera 13

Q-10. A young labor attended the eye OPD with complaints of a growth on the medial side of the
right eye, on examination a fleshy growth from the medial side of the conjunctiva was seen
encroaching on to the cornea
a) What is the diagnosis?
b) How would you treat such a case?

ANS:
a) PTERYGIUM
b) TREATMENT:
 Medical:
1. Tear substitute to avoid dryness
2. Topical steroids
3. The patient may be advised to wear sunglasses to reduce ultraviolet exposure in order
to decrease the growth stimulus.
 Surgical:
1. Simple excision (‘bare sclera’ technique) is associated with a high rate of recurrence
(around 80%), often with more aggressive behavior than the original lesion.
2. Simple conjunctival flap.
3. Conjunctival auto grafting, the donor conjunctival patch is usually harvested from the
superior or super temporal para-limbal the site generally heals rapidly, even without
suturing.
4. Amniotic membrane patch grafting is an alternative. Both conjunctival grafts and
amniotic membranes can be secured with tissue glue rather than sutured, shortening
operating time and reducing postoperative irritation.
5. Adjunctive treatment with mitomycin C
6. beta irradiation are sometimes used in place of patching techniques.
7. Peripheral lamellar keratoplasty may be required for deep lesions.
8. Argon laser to destroy regrowth of new vessels at limbus.

Q-11. A young patient came to the eye OPD with complaints or deteriorating vision for the last
few months He had three different pairs of glasses which were prescribed to him within one
year-on examination there was an irregular reflex on retinoscopy.
a) What is the probable diagnosis?
b) What other signs would be present?
c) What is the treatment?

ANS:
a) KERATOCONUS
b) OTHER SIGNS:
1. Oil Droplet Reflex on Opthalmoscopy
2. Keratomerty initially shows irregular astigmatism
3. Conical shape of cornea
4. Placido disc (Distortion of Corneal Reflex)
5. Munson’s Sign (Indentation or Acute Bulge of Lower Lid)
6. Upon Slit Lamp Examination:
i) Vogt’s Lines
ii) Fleischer Ring

KMU Past Papers Solved SEQs


Conjunctiva, Cornea and Sclera 14

iii) Edema and Opacity of Stroma


c) TREATMENT:
1. Spectacles and Soft Contact Lens (early stages)
2. Piggy Back Contact Lenses
3. Intrastromal insertion of Stromal ring of 0.25 – 0.45 mm thickness
4. Keratoplasty (penetrating or deep lamellar)

Q-12. A forty years Old gardener presented with history of a trauma with a vegetable matter.
After that he developed foreign body sensation, blurred vision and corneal infiltration.
a) What are the possible causes?
b) How will you treat it?

ANS:
a) FUNGAL KERATITIS
b) TREATMENT:

 Medical Treatment:
Improvement may be slow in comparison to bacterial infection.
1. General measures
i) Hospital admission should be considered for patients who are not likely to comply
or are unable to self-administer treatment.
ii) Discontinuation of contact lens wear is mandatory.
iii) A clear plastic eye shield should be worn between eye drop instillation if
significant thinning (or perforation) is present.
2. Removal of the epithelium over the lesion may enhance penetration of antifungal agents.
It may also be helpful to regularly remove mucus and necrotic tissue with a spatula.
3. Topical antifungals should initially be given hourly for 48 hours and then reduced as signs
permit. Because most antifungals are only fungistatic.
i) Candida infection is treated with amphotericin B, natamycin, fluconazole, and
clotrimazole.
ii) Filamentous infection is treated with natamycin, econazole, amphotericin B, and
miconazole.
4. A broad-spectrum antibiotic might also be considered to address or prevent bacterial co-
infection.
5. Cycloplegia (Atropine) as for bacterial keratitis.
6. Subconjunctival fluconazole may be used in severe cases
7. Systemic antifungals may be given in severe cases, when lesions are near the limbus, and
for suspected endophthalmitis.
8. Tetracycline (doxycycline) may be given for its anti-collagenase effect when there is
significant thinning.
 Surgical Treatment:
1. Debridement and Superficial keratectomy can be effective to de-bulk a lesion.
2. Therapeutic keratoplasty is considered when medical therapy is ineffective.
3. Anterior chamber washout with intra-cameral antifungal injection may be considered for
unresponsive cases in which there is a stable corneal infiltrate but enlarging endothelial
exudation.
4. Cauterization of ulcer
5. Conjunctival flap

KMU Past Papers Solved SEQs


Conjunctiva, Cornea and Sclera 15

Q-13. A nine-year-old male Child presents ever summer with intense itching. Photophobia and
foreign sensation examination there is papillary hypertrophy on superior tarus keratopathy of
both eyes and plaque on cornea of left eye.
a) What is diagnosis?
b) How will you treat it?

ANS:
a) VERNAL KERATO-CONJUNCTIVITIS
b) TREATMENT:
Already mentioned**

Q-14. A young boy of 18 years of age complains to his doctor of progressive decrease in vision.
The examining doctor noted irregular astigmatism with conical cornea.
a) What is the most probable diagnosis?
b) What signs would you like to see in such a patient to reach at the diagnosis?
c) How will you treat such a patient?

ANS:
a) KERATOCONUS
b) SIGNS:
1. Oil Droplet Reflex on Opthalmoscopy
2. Keratomerty initially shows irregular astigmatism
3. Conical shape of cornea
4. Placido disc (Distortion of Corneal Reflex)
5. Munson’s Sign (Indentation or Acute Bulge of Lower Lid)
6. Upon Slit Lamp Examination:
i) Vogt’s Lines
ii) Fleischer Ring
7. Edema and Opacity of Stroma
c) TREATMENT:
1. A Spectacles and Soft Contact Lens (early stages)
2. Piggy Back Contact Lenses
3. Intrastromal insertion of Stromal ring of 0.25 – 0.45 mm thickness
4. Keratoplasty (penetrating or deep lamellar)

Q-15. A 73 years old woman develops a painful rash on her right side of the face, forehead and nose
area. Her right eye is closed because of associated lid edema.
a) What is the most probable diagnosis?
b) Name the ocular lesions in this condition?
c) What is the treatment of this condition?

ANS:
a) HERPES ZOSTER OPTHALMITIS
b) OCULAR COMPLICATIONS:
1. Conjunctivitis,

KMU Past Papers Solved SEQs


Conjunctiva, Cornea and Sclera 16

2. Scleritis,
3. Episcleritis,
4. Keratitis
5. Iridocyclitis,
6. Argyll-robertson pupil,
7. Cranial Nerve Palsies
8. Acute Retinal Necrosis
9. Optic Neuritis
10. Corneal Opacity
c) TREATMENT PLAN:
1. Antiviral Drugs
i) Systemic – Oral Acyclovir
ii) Topical – Acyclovir 3% Eye Ointment
2. Analgesics and anti – inflammatory (Pethidine)
3. Topical Atropine (applied in cases of keratitis, iridocyclitis and scleritis)
4. Antibiotics (if epithelium is ulcerated or when topical steroids are used)
5. Corticosteroids
i) Topical Steroids (Used in Disciform Keratitis, Scleritis and Iridocyclitis)
ii) Systemic Steroids (Indicated in Cranial Nerve Palsies)
6. Antidepressants (Amitriptylene)
7. Artificial Tears and Lateral Tarsorrhaphy (Indicated for Neuroparalytic Ulcers in cases of
Dry Eye)
8. Full thickness keratoplasty

Q-16. A 13 years adolescent male patient complains of intolerance of light, sever itching, pain
redness and watering of eyes for the last eight years. The symptoms are severer in the spring
and summer season. He has been using medicines from quakes since the start of the disease.
He also complains of progressive decline in vision for the last three years. The best corrected
visual acuity is "(With - 7.00DS and -10.00DS at 120 degrees) in right eye and 6/36(with - 6.00
and -7.00DC at 60 degrees) in left eye. The eyes are red, there are large cobble-stone papillae
on the upper tarsal plate and large papillae around the limbus.
a) What are the two diseases that this patient is suffering from?
b) How will you treat each condition?
c) What are the other three most common complications that can arise from long standing use of
medicines prescribed by quakes in the condition?

ANS:
a) DISEASES:
1. Long Standing Vernal Kerato-conjunctivitis
2. Keratoconus
b) MENTIONED BEFORE**
c) COMPLICATIONS:
1. Steroid Induced Glaucoma
2. Cataract
3. Fungal and viral infections of the cornea

KMU Past Papers Solved SEQs


Conjunctiva, Cornea and Sclera 17

Q-17. Two weeks old child is brought to eye ward as an emergency the infant’s eyelids are
matted together with profuse purulent discharge
a) What is the diagnosis and enumerate the causes?
b) What is microbial investigation would you carry out
c) What is the treatment

ANS:
a) NEONATAL CONJUNCTIVITIS OR OPTHALMIA NEONATORAM
Causes
1. Virulent gonococcus
2. Chlamydia oculogenitalis,
3. Streptococcus pneumoniae
b) MICROBIAL INVESTIGATION:
1. Conjunctival scraping for Gram stain or Giemsa stain
2. Culture on chocolate agar and/or Thayer-Martin for N.Gonorrhea
3. Culture on blood agar for other bacteria
4. Polymerase chain reaction assay (PCR) to detect chlamydia and gonorrhea
c) TREATMENT:
1. The treatment prior to laboratory results should include topical erythromycin ointment and
an IV or IM third-generation cephalosporin.
2. This infection is treated with oral erythromycin (50 mg/kg/d divided qid) for 14 days.
3. Topical erythromycin ointment may be beneficial as an adjunctive therapy.
4. Parenteral Penicillin or Newer Cephalosporin

Q-18. An 18 year old female student nurse presents symptoms of severe grittiness, lacrimation,
and photophobia in both eyes. On examination she has bilateral chemosis, intense,
conjunctiva! injection, petechial hemorrhages but no purulent discharge. There are bilateral
conjunctival follicles with no papillae. The pre auricular lymph nodes are enlarged, her visual
acuities are 6/9 both eyes with normal papillary responses. The corneas and anterior
chambers are normal. After 1 week she develops bilateral corneal sub epithelial infiltrates.
Her visual acuities dropped to 6/12 both eyes, 2 of her room mates have also developed the
same eye.
a) What is the likely diagnosis?
b) What conditions can cause conjunctivitis with enlarged pre auricular lymph nodes and a
history of sexual contacts.
c) What is the treatment of this patients before she developed corneal infiltrates?
d) What is the treatment for keratopathy in this case?
e) Would you prescribe e topical antibiotics in this patients?
f) What advice would you give to the patient to prevent her from passing on the infection to
her patients, colleagues and family members?

ANS:
a) EPIDEMIC KERATO-CONJUNCTIVITIS
b) CONDITIONS:
1. Pharyngo-conjunctival Fever
2. Gonococcal Conjunctivitis
3. Adult Inclusion Conjunctivitis (a subtype of chlamydial conjunctivitis)
c) TREATMENT BEFORE CORNEAL INFILTRATES:

KMU Past Papers Solved SEQs


Conjunctiva, Cornea and Sclera 18

1. Artificial tears
2. Cold compresses
3. Cycloplegic agents for severe photophobia
4. Topical corticosteroids
5. Topical agents that have antiviral activity
d) TREATMENT OF KERATOPATHY:
1. Superficial debridement
2. Lamellar keratectomy
e) YES, to prevent secondary bacterial infection
f) ADVISE THE PATIENT
1. Avoided rubbing with hands,
2. Frequent hand washing
3. Use sunglasses
4. Avoid sharing daily use article with near and dear ones,
5. Give health education to family members about the disease.

Q-19. A 50 years old male presents with history of trauma RE with leaf of sugarcane five days
back. He is complaining of foreign body sensation, photophobia and blurring of vision. On
examination there is grayish stromal infiltrate, surrounded by satellite, feathery, finger like
lesions with small hypopyon.
a) Briefly discuss the most likely diagnosis.
b) Briefly discuss how you will manage this patient?

ANS:
a) FUNGAL KERATITIS
b) MENTIONED BEFORE**

Q-20. A 16 years old boy with a history of allergic conjunctivitis for about 6 years, presented
with gradual deterioration of vision on both eves. He is already using high minus glasses, but
not satisfied because there is frequent change in his glasses number. On torch Light
examination both cornea look clear but conical.
a) What is the most probable diagnosis?
b) Name two ocular and two systemic associations of this condition?
c) What are the steps of management?

ANS:
a) KERATOCONUS
b) OCULAR ASSOCIATION:
1. Blue Sclera
2. Ectopic Lentis
c) SYSTEMIC ASSOCIATION:
1. Down’s Syndrome
2. Ehlers – Danlos Syndrome
d) MENTIONED BEFORE

KMU Past Papers Solved SEQs


Conjunctiva, Cornea and Sclera 19

Q-21. A gardener is working in a garden trimming the hedge when some of the trimmings went
into his left eye. He washed his eye with tap water. His eye gradually became sore. 2 days later
he presented at the ward as an emergency. On examination VA Right 6/6 Left 6/12. Left eye
Circum-Corneal injection. 2xI mm white corneal lesion, feathery appearance at 4 0'clock
position. Occasional cells in anterior chamber.
a) What is the likely diagnosis?
b) What are the commonest organisms?
c) What microbiological investigations are you going to order?
d) What topical treatment are you going to advise?
e) What are the sight threatening complications of Suppurative Keratitis?
f) What would you tell the patient about the prognosis of his problem?

ANS:
a) FUNGAL KERATITIS
b) COMMONEST ORGANISMS:
1. Candia albicans
2. Aspergillus fumigatus
3. Fusarium
4. Cephalosporium
5. Streptothrix actinomycosis
c) MICROBIOLOGICAL INVESTIGATIONS:
1. Staining with Methamine Silver, Gram and Gimesa Stains
2. Culture in Sabouraud’s medium, blood – agar plate or brain – heart infusion broth is
essential
d) TOPICAL TREATMENT:
1. Natamycin (5%) Eyedrops is instilled 1 hourly
2. Miconazole (1%) eye ointment is applied 5 times daily
3. Nystatin eye ointment is applied 5 times daily
4. Topical Amphoteracin B (0.25%) is instilled 1 hourly
e) SIGHT THREATENING COMPLICATIONS:
1. Decementocele
2. Perforation of Corneal Ulcer leading to sub-laxation of lens
3. Endopthalmitis or Retinal Detachment
4. Corneal Scaring
5. Toxic Iridocyclitis
f) PROGNOSIS: 2 – 3 WEEKS

Q-22. A lady age 40 years present to you with a red eye. What import signs and symptoms will
you look for to establish a diagnosis?

ANS:
Symptoms to ask and Signs to look for in Red Eye:
1. History of trauma
2. Onset of symptoms
3. Pain
4. Effect on vision
5. Discharge from eyes

KMU Past Papers Solved SEQs


Conjunctiva, Cornea and Sclera 20

6. Congestion of Eye structures


7. Associated symptoms i.e. Nausea, vomiting
8. On examination look for Corneal Haziness, Iris, Pupil
9. Tenderness of orbit
10. Clarity of anterior chamber
11. Examine by Slit Lamp for Anterior Chamber, Iris, Foreign Body, Closed Angle
12. Check Visual Acuity
13. Digital Tonometry
14. Gonioscopy (To see angle for Angle Closure Glaucoma)

Q-23. A 70 years old diabetic man complaining of pain on one side of the forehead for three days.
There is also rash on that side of the forehead. The rash is also involving the side of the nose.
a) What is the most probable diagnosis?
b) What are the 8 common ocular complications of this condition?
c) What is the treatment of this patient?

ANS:
a) HERPES ZOSTER OPTHALMITIS
b) OCULAR COMPLICATIONS:
1. Occulomotor Nerve Paralysis
2. Abducent Nerve Paralysis
3. Facial Nerve Paralysis
4. Exposure Keratitis
5. Acute Retinal Necrosis
6. Optic Neuritis
7. Corneal Opacity
8. Iridocyclitis and Scleritis
c) TREATMENT PLAN:
1. Antiviral Drugs
i) Systemic – Oral Acyclovir
ii) Topical – Acyclovir 3% Eye Ointment
2. Analgesics and anti – inflammatory (Pethidine)
3. Topical Atropine (applied in cases of keratitis, iridocyclitis and scleritis)
4. Antibiotics (if epithelium is ulcerated or when topical steroids are used)
5. Corticosteroids
i) Topical Steroids (Used in Disciform Keratitis, Scleritis and Iridocyclitis)
ii) Systemic Steroids (Indicated in Cranial Nerve Palsies)
6. Antidepressants (Amitriptylene)
7. Artificial Tears and Lateral Tarsorrhaphy (Indicated for Neuroparalytic Ulcers in cases of
Dry Eye)
8. Full thickness keratoplasty

KMU Past Papers Solved SEQs


Conjunctiva, Cornea and Sclera 21

Q-24. A mother brings a 6 months old male child to the eye OPD with the complaints of
photophobia and lacrimation. Ocular examination revealed large hazy Cornea.
a) What are the differential diagnoses?
b) What is the most probable cause?
c) How will you treat this child?

ANS:
a) DIFFERENTIAL DIAGNOSIS:
1. Congenital glaucoma
2. Nasolacrimal Duct blockage
3. CHED
4. Forceps delivery damage
b) CONGENITAL GLAUCOMA
c) TREATMENT:
 Medical
i) Systemic Acetazolamid and Mannitol I/V along with Local Beta Blockers
 Surgical Treatment:
1. Goniotomy
2. Trabeculotomy
3. Trabeculectomy

KMU Past Papers Solved SEQs


Glaucoma 22

Q-1. A 60 years old women presents with sudden loss of vision in left eye with pain and
vomiting. The visual acuity was count finger in the left eye. The eye showed diffuse corneal
haze and shallow anterior chamber.
a) What is most likely diagnosis?
b) What are the risk factors for the disease?
c) Give treatment?

ANS:
a) ACUTE ANGLE CLOSURE GLAUCOMA
b) RISK FACTORS:

Anatomical Risk Factors:


It is typically seen in eyes which are:
1. Small and hypermetropic eye
2. Anatomical narrow angle of the anterior chamber
3. Shallow anterior chamber
4. Iris—lens diaphragm is pushed forwards.
General Risk Factors:
1. Age: It affects mainly in the 5th-6th decade
2. Sex: Women are usually affected (male: female ratio is 1:4)
3. It is usually bilateral but one eye is involved first
4. Personality: Highly strung, anxious persons with unstable vasomotor system
5. Race: It is common among Asians and Eskimos but rare in Africans and Caucasians
6. Genetic factors: They are important but poorly defined, with an increased prevalence of
angle closure in family members.
c) MANAGEMENT:
Initial treatment
1. The patient should assume a supine position to encourage the lens to shift posteriorly
under the influence of gravity.
2. Acetazolamide 500 mg is given intravenously if IOP >50 mmHg, and orally if IOP is <50
mmHg.
3. Contraindications include sulfonamide allergy and angle closure secondary to
topiramate/other sulfa derivatives).
4. A single dose of each of apraclonidine 0.5% or 1%, timolol 0.5%, and prednisolone 1% or
dexamethasone 0.1% to the affected eye, leaving 5 minutes between each.
5. Pilocarpine 2–4% one drop to the affected eye, repeated after half an hour; one drop of
1% into the fellow eye.
6. Analgesia and an antiemetic may be required.
Resistant cases
1. Central corneal indentation with a squint hook or indentation goniolens to force aqueous
into the angle; epithelial oedema can be cleared first with topical 50% glycerol to improve
visualization and avoid abrasion.
2. Further pilocarpine 2–4%, timolol 0.5%, apraclonidine 1% and topical steroid.
3. Mannitol 20% 1–2 g/kg intravenously over 1 hour, oral glycerol 50% 1 g/kg, or oral
isosorbide 1–1.5 g/kg, having checked for contraindications.
4. Early laser iridotomy or iridoplasty after clearing corneal oedema with glycerol.
5. Paracentesis can be performed, but carries significant risks.
6. Surgical options: peripheral iridectomy, lens extraction, goniosynechialysis,
trabeculectomy and cyclodiode.

KMU Past Papers Solved SEQs


Glaucoma 23

Bilateral laser iridotomy is performed once an attack has been broken, signified by a clear
cornea and preferably normalized IOP. Topical steroids and any necessary hypotensives are
continued for at least a week.

Q-2. A 50 years old female comes for routine checkup. On examination her VA is 6/6 each eye.
Intra ocular pressure is 35 mmHg each eye. Cup disk ratio 0.7 each eye.
a) What is the diagnosis?
b) How will you evaluate such a patient?
c) How will you treat this patient?

ANS:

a) PRIMARY OPEN ANGLE GLAUCOMA (POAG)


b) EVALUATION OF PATIENT:
 History
1. Visual symptoms will usually be absent, unless damage is advanced. Sometimes symptomatic
central field defects may occur at an early stage, in the presence of a relatively normal
peripheral field.
2. Previous ophthalmic history
i) Refractive status as myopia carries an increased risk of POAG.
ii) Causes of secondary glaucoma such as ocular trauma or inflammation; previous eye
surgery, including refractive surgery, may affect IOP readings.
3. Family history
i) POAG or related conditions such as OHT. (ocular hypertension)
ii) Other ocular disease in family members.
4. Past medical history
i) Asthma, heart failure or block, peripheral vascular disease: contraindications to the use of
beta-blockers.
ii) Head injury, intracranial pathology including stroke: may cause optic atrophy or visual field
defects.
iii) Vasospasm: migraine and Raynaud phenomenon.
iv) Diabetes, systemic hypertension and cardiovascular disease may increase the risk of POAG.
v) Oral contraceptive pill for several years may be associated with an increased risk of
glaucoma.
5. Current medication
i) Steroids including skin cream and inhalants.
ii) Oral beta-blockers may lower IOP.

KMU Past Papers Solved SEQs


Glaucoma 24

6. Social history including smoking and alcohol intake, especially if toxic/nutritional optic
neuropathy is suspected.
7. Allergies, particularly to any drugs likely to be used in glaucoma treatment, e.g. sulfonamides.
 Examination
1. Visual acuity is likely to be normal except in advanced glaucoma.
2. Pupils. Exclude a relative afferent pupillary defect (RAPD); if initially absent but develops later,
this constitutes an indicator of substantial progression.
3. Colour vision assessment such as Ishihara chart testing if there is any suggestion of an optic
neuropathy other than glaucoma.
4. Slit lamp examination. Exclude features of secondary glaucomas such as pigmentary and
pseudoexfoliative.
5. Tonometry prior to pachymetry, noting the time of day.
6. Gonioscopy. For angle closure assessment.
7. Optic disc examination for glaucomatous changes, should always be performed with the
pupils dilated, provided gonioscopy does not show critically narrow angles. Red-free light can
be used to detect RNFL (retinal nerve fibre layer) defects.
 Investigation
1. Pachymetry for CCT (central corneal thickness)
2. Perimetry should usually be performed prior to clinical examination.
3. Imaging of the optic disc, peripapillary RNFL (retinal nerve fibre layer) and/or ganglion cell complex,
e.g. red-free photography, stereo disc photography, OCT, confocal scanning laser
ophthalmoscopy and/or scanning laser polarimetry.
c) TREATMENT:
The main aim of treatment is to prevent visual loss and visual field defect which result from high
intraocular pressure.
Methods
The treatment options available at present are medicines, laser or surgery to lower the
intraocular pressure.
1. Medical: It is always the treatment of choice in the early stages. (Local, Systemic, Hyperosmotic
Agents or Combined)
2. Surgical: It is considered to be the last resort.
3. Argon or diode laser trabeculoplasty (ALT or DLT): It is the most advanced technique.
4. Recent advanced procedures: These include
i) Laser filtration,
ii) Seton valves,
iii) Deep sclerotomy and
iv) Viscocanalostomy

Q-3. A 45 years old lady presents with sudden loss of visual in her right eye for last 2 days she
also complains of severe pain headache and vomiting. On examination there is circum-corneal
congestion, corneal edema and raised IOP in her right eye left eye is normal.
a) Discuss the most likely diagnosis?
b) What are differential diagnosis?
c) How will you manage this patient?

ANS:
a) ACUTE ANGLE CLOSURE GLAUCOMA
b) DIFFERENTIAL DIAGNOSIS:
1. Acute Iridocyclitis
2. Bacterial Keratitis
3. Optic neuritis

KMU Past Papers Solved SEQs


Glaucoma 25

4. Endopthalmitis
5. Acute corneal hydrops as in keratoconus
c) MANAGEMENT:
Already Mentioned **

Q-4. A 45 years old lady presents with sudden RE visual loss associated with period congestion,
nausea and vomiting for the last 3 days. On examination there is circum-corneal edema with
epithelial vesicles. The pupil is vertically oval, fixed and in Her IOP is 50 mmHg Right Eye and
12 mmHg Left Eye.
a) Discuss the most likely diagnosis?
b) How will you manage this patient?

ANS:
a) ACUTE ANGLE CLOSURE GLAUCOMA IN RIGHT EYE
b) MANAGEMENT:
Already mentioned**

Q-5. A 45 year lecturer from the Agricultural University request an urgent consultation with an
Ophthalmologist for a second opinion. He is very worried because he has been told by another
ophthalmologist that he has Glaucoma and he has already been commenced on anti-Glaucomal
medications. His Visual acuities are 6/6 with appropriate glasses. There is NO pupillary deficit.
Intraocular pressures are 20mmHg. The anterior Chambers are deep, with wide open angles.
There is no anterior segment inflammation. The optic discs do NOT Show any cupping. His
Central Corneal thickness is 625 Microns in both eyes. His anti-glaucomal medications were
stopped in 2 weeks on repeat IOP Check the intraocular Pressure are 26mmHg in each eye. His
automated Visual filed analysis does not show any defect. Ocular Coherence Tomography of
both Optic nerves is normal.
a) What Ocular condition does this patient have?
b) Does every intraocular Pressure of 26mmHg need anti Glaucoma medication?
c) Enumerate Clinical features of Optic disc damage in Glaucoma?
d) Enumerate classes of Anti Glaucoma drugs currently used in POAG?
e) What questions would you ask a patient of Glaucoma before prescribing Topical Beta
Blockers?

ANS:
a) NORMAL EYE
b) No, IOP depends on corneal thickness, normal corneal thickness is 540 microns. If the cornea is
thinner, an underestimation of IOP is likely to result, and if thicker, an overestimation. Corneas
tend to be thicker than average in individuals with ocular hypertension, and thinner in normal-
tension glaucoma (NTG); here in this case it is 625 microns, 75 microns above the normal range
and every 20 microns increase in corneal thickness will show false positive result of 1mmhg IOP,
so the patient will have IOP of 26-5 = 21mmhg which is normal.
c) CLINICAL FEATURES OF OPTIC DISC DAMAGE IN GLAUCOMA:
1. Optic nerve atrophy (cavernous optic atrophy)
2. Cup Disc Ration vary from 0.6 to even 0.9 (in vertical axis)
3. Asymmetry in the Optic Nerve Heads of more than 0.2
4. Laminar Dot Sign (Dot or Slit like opening of the Lamina cribrosa)
5. Bayoneting sign (the retinal vessels appear to be broken off at the margin of the cup)

KMU Past Papers Solved SEQs


Glaucoma 26

6. Nasalization of blood vessels


7. Splinter Hemorrhages at disc margins
d) CLASSES OF ANTI – GLAUCOMA DRUGS USED IN POAG:
1. Miotics/Parasympathomimetics (Pilocarpine)
2. Sympatholytic Drugs (Timolol, Betaxolol)
3. Sympathomimetic Drugs (Epinephrine, Brimonidine)
4. Prostaglandin and Prostamide Analogues (Latanoprost, Travoprost)
5. Carbonic Anhydrase Inhibitors (Dorzolamide, Brinzolamide)
e) QUESTIONS ASKED BEFORE PRESCRIBING TOPICAL BETA BLOCKERS:
1. Does the patient suffer from Asthma?
2. Does the patient suffer from any Cardiovascular Disease?
3. Does the patient have a history of Hypotensive episodes?
4. Does the patient suffer from any peripheral vascular disease?

Q-6. A 60-year-old woman presented to the ophthalmologist because of several episodes of


sharp pain in her right eye over the past few months her visual acuity was normal in both eyes
and the intraocular pressures were normal. Slit lamp examination showed a shallow anterior
Chamber.
a) What is the diagnosis?
b) What Other physical signs may be present?
c) What is the treatment?

ANS:
a) PRIMARY ANGLE CLOSURE GLAUCOMA (SUBACUTE)
b) PHYSICAL SIGNS:
1. Gonioscopy
i) Shows narrow angle of the anterior chamber grade grade 1 – 2.
ii) Occasional peripheral anterior synechiae may be present.
iii) There is narrow angle recess with clumping of pigments in the angle
2. Provocative test is positive.
3. Perimetery to assess visual field defect if any
c) Treatment:
During the attack:
Pilocarpine is used
After the attack:
A prophylactic communication is made between the posterior and anterior chamber by
making hole in the iris by
1. Peripheral Laser Iridotomy in both eyes
2. Peripheral Surgical Iridectomy in both eyes

Q-7. A 50 years old female patient presented in OPD with ocular pain headache blurred vision
and rainbow colored halos around her eyes accompanied by nausea and vomiting. On
examination there was corneal edema mid dilated and sluggish pupils with iris Bombe.
a) What two ocular examinations would you like to do for definitive diagnosis?
b) What is treatment?
c) Write three differential of given scenario.

ANS:

KMU Past Papers Solved SEQs


Glaucoma 27

a) OCULAR EXAMINATIONS:
1. Gonioscopy
2. Provocative test
3. Tonometry
b) TREATMENT:
During the attack:
Pilocarpine is used
After the attack:
A prophylactic communication is made between the posterior and anterior chamber by
making hole in the iris by
1. Peripheral Laser Iridotomy in both eyes
2. Peripheral Surgical Iridectomy in both eyes

c) DIFFERENTIAL DIAGNOSIS:
1. Acute Angle Closer Glaucoma
2. Acute Iridocyclitis
3. Acute Endopthalmitis
4. Acute Kerato-conjunctivits

Q-8. A 40-year-old lady with a refractive error of +4 diopters in each eye presents with
episodes of blurred vision halos and mild pain especially in the evening. The symptoms are
relieved with sleep, her visual acuities are 6/6, both eyes. The anterior chamber depth is 2.2
mm and both anterior segments are quiet. There is no RAPD.
a) What is the differential diagnosis?
b) What further clinical examination of anterior segment is warranted?
c) What are the signs of acute angle closure glaucoma?
d) What class of drugs are commonly used in treatment of acute closure glaucoma and why?

ANS:
a) SUB-ACUTE ANGLE CLOSURE GLAUCOMA
b) CLINICAL EXAMINATION
1. Anterior segment Examination
2. Gonioscopy which show narrow angle
3. Perimetery to assess visual field defect if any
c) SIGNS OF ACUTE ANGLE CLOSURE GLAUCOMA:
1. Decreased visual acuity
2. Conjunctival redness and circumcorneal congestion
3. Edematous cornea
4. Pupil is vertically oval fixed in semi-dilated position un-responsive to both light and
accommodation
5. Iris surface show atrophic surface
6. Optic disc swelling
7. Raised IOP
d) CLASSES OF DRUGS USED:
1. Miotics/Parasympathomimetics (Pilocarpine)
2. Sypatholytic Drugs (Timolol, Betaxolol)
3. Carbonic Anhydrase Inhibitors (Dorzolamide, Brinzolamide)

KMU Past Papers Solved SEQs


Glaucoma 28

Q-9. A 70 years old man presented with dimness of vision in both eyes for the last six months.
His visual acuity is 6/36 both eyes. The IOP was 26mmHg in both eyes. Fundi showed a Cup
Disc ratio of 0.5 in right eye and 0.8 in left eye.
a) What is your diagnosis?
b) What are your differential diagnoses?
c) What are the risk factors for this disease?
d) How you treat such a case?

ANS:
a) PRIMARY OPEN ANGLE GLAUCOMA
b) DIFFERENTIAL DIAGNOSIS:
1. Bilateral Cataract
2. Age Related Macular Degeneration
3. Chorio-retinal Degeneration
4. Optic Atrophy
5. Corneal Dystrophy
6. Corneal Degeneration e.g. band keratopathy
7. Diabetic Retinopathy
8. Refractive Errors
c) RISK FACTORS:
1. Advancing Age
2. Myopia
3. Diabetes
4. Family History
5. Prolonged Steroids use
d) TREATMENT:
 Medical treatment:
1. Beta blockers
2. Alpha 2 adrenergic antagonists
3. Carbonic anhydrase inhibitors
4. Meiotic
5. Adrenergic drugs
6. Prostaglandins
 Laser Treatment:
1. Argon Laser Trabeculoplasty
2. Diode Laser Trabeculoplasty
 Filtration Surgery:
1. Trabeculectomy

Q-10. A 65 years old lady complains of painful red eye for last 5 days. She had a visual loss since
one year in that eye. On examination she has a corneal edema, shallow anterior chamber, a
hyper mature cataract and IOP of 50 mmHg.
a) What are the differential diagnoses?
b) What is the most probable diagnosis?
c) What is the emergency treatment?
d) What is the definitive treatment?

ANS:

KMU Past Papers Solved SEQs


Glaucoma 29

a) DIFFERENTIAL DIAGNOSIS:
1. Glaucoma as a Cataract Complication (Phaco-antigenic Glaucoma, Phacolytic glaucoma and
hyper mature cataract phacomorphic glaucoma)
2. Absolute Glaucoma
3. Acute Congestive Glaucoma
b) PHACOLYTIC GLAUCOMA
c) EMERGENCY TREATMENT:
Decrease Intraocular Pressure
d) DEFINITIVE TREATMENT:
Trabeculectomy

Q-11. A 2 months male baby is brought to you with watering and discharge from both eyes since
birth.
a) What are the differential diagnoses?
b) What is the most likely diagnosis?
c) What are treatment options in such a case?

ANS:
a) DIFFERENTIAL DIAGNOSIS:
1. Congenital glaucoma
2. Congenital Nasolacrimal Duct Blockage
3. Ophthalmia neonatorum
b) PRIMARY CONGENITAL (INFANTILE) GLAUCOMA
c) TREATMENT:
 Medical Treatment:
1. Systemic Acetazolamide and Mannitol I/V along with Beta Blockers to control Intraocular
Pressure
 Surgical Treatment:
1. Goniotomy
2. Trabeculotomy
3. Trabeculectomy

Q-12. A 55 years old cinema worker presents in the casualty with Nausea, severe headache and
pain in the left eye since last night. He is covering his left eye. On examination: Right eye VA
6/6, Right eye normal except narrow angle on Gonioscopy Intra ocular pressure 16mm Hg.
Left eye VA 6/36 has Ciliary Flush intraocular pressure 60 mmHg Anterior chamber is shallow
and Hazy Pupil semi dilated and poorly reacting to light Corneal Haze and Edema.
a) What is the condition and why?
b) What is the initial management of this condition and why?
c) How would you protect the fellow eye from this condition?

ANS:
a) ACUTE CONGESTIVE GLAUCOMA
Reasons:
1. Hyper Acute Onset
2. Narrow angle of fellow normal eye
3. Raise IOP of Left Eye

KMU Past Papers Solved SEQs


Glaucoma
Uvea, Lens and Vitreous 30

4. Shallow Anterior Chamber


5. Semi-dilated Pupil not reacting to light
6. No discharge or pus
b) INITIAL MANAGEMENT:
Take the patient to well illuminated room and lay him in bed in supine position
Reduce intraocular pressure
c) MANAGEMENT OF THE FELLOW EYE:
Laser Irodotomy or use of Sun glasses/shades

Q-13. A 45 years old lady presents with Sudden Right Eye visual loss associated with period
congestion, nausea and vomiting for the last 3 days. On examination there is circumcorneal
edema with epithelial vesicles. The pupil is vertically oval, fixed and in Her IOP is 50 mmHg in
Right Eye and 12 mmHg Left Eye.
a) Discuss the most likely diagnosis?
b) How will you manage this patient?

ANS:
a) ACUTE ANGLE CLOSER GLAUCOMA
b) MANAGEMENT:
Already mentioned**

KMU Past Papers Solved SEQs


Uvea, Lens and Vitreous 31

Q-1. A 55 years old diabetic patient presented to the doctor after 2 days of cataract surgery. He
complained of increasing pain, decreased vision and lid swelling. He admitted an initial
improvement on first postoperative day. His visual in hand movement.
a) Briefly discuss the most likely diagnosis
b) Differential Diagnosis?
c) How will you manage this patient?

ANS:
a) ACUTE POSTOPERATIVE ENDOPHTHALMITIS
b) DIFFERENTIAL DIAGNOSIS:
1. Retained lens material in the anterior chamber.
2. Vitreous hemorrhage, especially if blood in the vitreous is depigmented.
3. Postoperative uveitis.
4. Toxic reaction to the use of inappropriate or contaminated irrigating fluid or viscoelastic.
5. Complicated or prolonged surgery may result in corneal oedema and uveitis.
c) MANAGEMENT:
1. Intravitreal antibiotics are the key to management. Antibiotics commonly used in
combination are ceftazidime, and vancomycin.
2. Subconjuctival Antibiotic Injections (amikacin if penicillin allergic)
3. Topical Antibiotics are of limited benefit and are often used only 4–6 times daily in order
to protect the fresh wounds from contamination.
4. Oral antibiotics Fluoroquinolones penetrate the eye well and moxifloxacin 400 mg daily
for 10 days is recommended;
5. Oral Steroids The rationale for the use of steroids is to limit destructive complications of
the inflammatory process.
6. Periocular steroids Dexamethasone or triamcinolone should be considered if systemic
therapy is contraindicated.
7. Topical Dexamethasone for anterior uveitis.
8. Topical Mydriatic such as atropine.
9. Intravitreal steroids may reduce inflammation in the short term but do not influence the
final visual outcome
10. Pars Plana Vitrectomy

Q-2. A 35 years old female presents with decreased vision and photophobia. On examination
her visual acuity is Counting Fingers both eyes, mutton fat keratic Precipitates (KPs), aqueous
cells, aqueous flare in both eyes and Dallen's Funch's Nodules in the right fundus oculi. She
gives history of penetrating trauma to right eye ten months back and corneo-sclarel repair
was done under General Anesthesia (GA).
a) What is the most likely diagnosis?
b) What are the differential diagnoses?
c) How will you treat this patient?

ANS:
a) ANTERIOR UVEITIS (GRANULOMATOUS)
b) DIFFERENTIAL DIAGNOSIS:
1. Iridocyclitis
2. Endopthalmitis
3. Sympathetic opthalmitis

KMU Past Papers Solved SEQs


Uvea, Lens and Vitreous 32

4. Posner–Schlossman syndrome (PSS)


5. Vogt–koyanagi–harada (vkh) syndrome

c) TREATMENT:
1. Local:
i) Hot Compress
ii) Dark Glasses or Eye shades
iii) Atropine 1% Eye drops or Ointment
iv) Tissue plasminogen activator (TPA)
v) Subconjunctival steroid (betamethasone sodium phosphate solution)
vi) Sub-Tenon injection
vii) Intraocular steroid injection (triamcinolone acetonide)
2. Systemic:
i) Corticosteroids eg. Prednisolone, Dexamethasone
ii) Analgesics and Anti-inflammatory Drugs
iii) Antibiotics
iv) Antimetabolites
3. Treatment of Complications:
i) Secondary Glaucome – Timolol maleate, betaxolol Eye drops
ii) Annular (ring) Synechiae and Iris Bombe – Laser Iridotomy

Q-3. A 30 year old gentleman of slightly dark complexion who is otherwise fit develops sudden
decrease in vision of the Left eye. His Visual acuities are 6/6 and Counting Fingers Right and
left Eye respectively. The Visual Acuity does not improve with pin hole in the Left eye. He has
a mild Left Relative Afferent Pupillary Defect (RAPD). The intraocular pressures are 18mmHg
in both eyes. Slit Lamp examination of Anterior Segment reveals iris neovascularization in the
left eye and a normal Right eye Anterior Segment. There are no lenticular opacities in either
eye. On the distant direct ophthalmoscopy there is no red reflex in the left eye with normal
reflex in the right eye there in no view of the Left Retina on indirect Ophthalmoscopy but
shows "ghost vessels" in the periphery of the right retina.
a) What is the likely cause of loss of red reflex in the Left eye?
b) How does one clinically examine structure of the eye posterior to the crystalline lens?
c) What is the imaging test of choice for the determining the cause of loss of red reflex?
d) What are the sight threatening sequel of ocular neovascularization?

ANS:
a) VITREOUS HEMORRHAGE
b) BY USING AN OPTHALMOSCOPE
c) B SCAN ULTRASONOGRAPHY is helpful in identifying fibrovascular proliferations on the retinal
surface and associated tractional or rhegmatogenous retinal detachment
d) SEQUEL
1. Vitreous hemorrhage
2. Neovascular glaucoma
4. Tarsorrhaphy—It is done in cases of neuroparalytic ulcers and exposure keratitis

KMU Past Papers Solved SEQs


Uvea, Lens and Vitreous 33

Q-4. A 20-year-old female presents with Acute onset of Severe pain, photophobia. Lacrimation
in the Right eye. Her vision is progressively getting worse in the affected eye. She also
complains of low back ache for the last 5 years and has difficulty bending even though her
abdominal girth is normal on examination her Right eye is red with a vision of 6/18. The
vision does not improve with pin hole. The Right pupil is imposed and fixed but the left pupil
is reacting to light on slit lamp examination she has marked circumcorneal injection, keratin
precipites, numerous cell with fibrin and hypopyon in addition there is posterior syncline
there are few cells in the anterior vitreous but hazy fundal details the IOP is normal in right
eye.
a) What is the diagnosis
b) What other ocular condition have the same presentation
c) What spinal disease can commonly can cause such an eye condition
d) If this patient has non-infectious intraocular inflammation what treatment would you gave
her
e) Enumerate the ocular causes of decreased vision in non-infectious intraocular
inflammation posterior to lens.

ANS:
a) ACUTE IRIDOCYLITIS
b) DIFFERENTIAL DIAGNOSIS:
1. Acute Angle Closure Glaucoma
2. Bacterial Keratitis
3. Endopthamitis
4. Sympathetic opthalmitis
c) ANKYLOSING SPONDYLITIS
d) TREATMENT:
1. Rest to the eye is given by dilatation of the pupil with 1% atropine.
2. Dark glasses or an eyeshade may also be used to avoid glare, discomfort and lacrimation
especially in sunlight.
3. Heat application improves blood circulation and relieves pain.
4. Control of acute phase of the inflammation with corticosteroids.
i) Topical Eyedrops and eye ointment, (dexamethasone)
ii) Subconjunctival injection, (betamethasone)
iii) Periocular injection of depot steroids (triamcinolone)
iv) Systemic steroids full course with maintenance tapering doses, (prednisolone)
5. Analgesics and anti-inflammatory relieve pain and discomfort, e.g. aspirin, ibuprofen, etc.
6. Modern broad-spectrum antibiotics which cross the blood-aqueous barrier are given in cases
of Infections.
7. Non-steroidal Anti-inflammatory (NSAIDs) and Cytotoxic Drugs (cyclosporine)
8. Specific treatment of the underlying disease should be added if the etiology is identified e.g.
Reiter’s syndrome, syphilis, tuberculosis etc.
e) CAUSES:
1. Choroiditis
2. Chorioretinitis
3. Endopthalmitis
4. Retinochoroiditis
5. Optic neuritis
6. Neuro-uveitis

KMU Past Papers Solved SEQs


Uvea, Lens and Vitreous 34

Q-5. A 65 years old patient presented with onset of pain, redness and photophobia. Associated
with decreased vision. On slit lamp examination fine keratic precipitate and fibrin were noted
on corneal endothelium. The anterior chamber showed cells flare and posterior synechiae.
there is also a history of low backache for last couple of months.
a) What is the most probable clinical diagnosis?
b) What investigation are needed for establishing etiological diagnosis
c) What is the management?

ANS:
a) ACUTE IRIDOCYLITIS
b) INVESTIGATION:
1. Hematological investigations:
i) Total and differential white blood cell count (TLC and DLC) is done to have a general
information about inflammatory response of the body.
ii) ESR to ascertain existence of any chronic inflammatory condition.
iii) Blood sugar levels to rule out diabetes mellitus.
iv) Blood uric acid in patients suspected of having gout.
v) Serological tests for syphilis, toxoplasmosis, and histoplasmosis.
vi) Tests for antinuclear antibodies, rheumatoid factor, C-reactive proteins
2. Urine examination is done for WBC, pus cells, RBC and culture to rule out urinary tract
infections.
3. Stool examination is done for cyst and ova to rule out parasitic infestations.
4. Radiological investigations include X-rays of chest, paranasal sinuses, sacroiliac joints and lumbar
spine.
5. Skin test: These include tuberculin test, Kveim’s test (sarcoidosis) and toxoplasmin test.

c) TREATMENT:
1. Rest to the eye is given by dilatation of the pupil with 1% atropine. (paralysis ciliary body
and iris, diminishes hyperaemia and prevents formation of posterior syneciae)
2. Dark glasses or an eyeshade may also be used to avoid glare, discomfort and lacrimation
especially in sunlight.
3. Heat application improves blood circulation and relieves pain.
4. Control of acute phase of the inflammation with corticosteroids.
i) Topical Eyedrops and eye ointment, (dexamethasone)
ii) Subconjunctival injection, (betamethasone)
iii) Periocular injection of depot steroids (triamcinolone)
iv) Systemic steroids full course with maintenance tapering doses, (prednisolone)
5. Analgesics and anti-inflammatory relieve pain and discomfort, e.g. aspirin, ibuprofen, etc.
6. Modern broad-spectrum antibiotics which cross the blood-aqueous barrier are given in
cases of Infections.
7. Non-steroidal Anti-inflammatory (NSAIDs) and Cytotoxic Drugs (cyclosporine)
8. Specific treatment of the underlying disease should be added if the etiology is identified
e.g. Reiter’s syndrome, syphilis, tuberculosis etc.

KMU Past Papers Solved SEQs


Uvea, Lens and Vitreous 35

Q-6. A 10 years old male child is brought to the eye OPD by his parents with the complaint of
not seeing the blackboard and TV from distance. He has no headaches.
a) What is the most probable diagnosis?
b) What are the types?
c) What are the treatment options?

ANS:
a) MYOPIA
b) TYPES:
1. Congenital Myopia
2. Simple Myopia
3. Pathological Myopia
4. Degenerative Myopia
c) TREATMENT OPTIONS:
Non-surgical
Use of Spectacles
Surgical
1. Radial Keratotomy
2. Excimer Laser
3. Epikeratophakia
4. Keratomileusis
5. LASIK (Laser assisted in situ Keratomileusis)

Q-7. An old man presented with a history of gradual decrease in the left eye for the past 6
months. There is no pain or any other symptoms. On distant direct ophthalmoscopy there is
dull red reflex in the center. All other structured are normal
a) What is the most common diagnosis?
b) How would you assess the retinal functions?
c) What are the methods of treatment and which is the most preferred one?

ANS:
a) CATARACT
b) ASSESSMENT OF RETINAL FUNCTION:
1. B scan ultrasonography to assess the eye posterior to lens
c) TREATMENT:
The type of operation depends on the individual case. In elderly persons the nucleus is hard and it
can be removed by the following methods:
1. Intracapsular cataract extraction (ICCE)—It has become obsolete nowadays.
2. Extracapsular cataract extraction (ECCE)
3. Phacoemulsification
4. Phacolysis
Most preferred one:
In the modern age ECCE is considered as the procedure of choice

KMU Past Papers Solved SEQs


Uvea, Lens and Vitreous 36

Q-8. A 75 years old male patient presents with bilateral progressive decrease in vision in both
of his eyes. He is non diabetic and non-hypertensive, rather his blood pressure is persistently
on the lower side. His anterior segment is normal. His intra ocular pressure is 14mm of Hg in
both his eyes. Vision is 6/36 in both eyes.
a) Enumerate three common diagnoses.
b) Enumerate three common investigations in this case.
c) Enumerate four long term complications of cataract extraction.

ANS:
a) COMMON DIAGNOSIS:
1. Bilateral Cataract
2. Age Related Macular Degeneration (ARMD)
3. Keratoconus
b) INVESTIGATIONS:
1. Distant Direct Ophthalmoscopy
2. Slit Lamp Examination
3. Fundus Fluorescein Angiography (FFA)
c) LONG TERM COMPLICATIONS:
1. Aphakia
2. Retinal Detachment
3. Opacification of Posterior Capsule
4. Chronic Endophthalmitis

Q-9. A 27 years old male patient with chronic backache comes to an eye OPD with photophobia,
redness and dimness of vision of the tight eye. On examination you can see whitish exudates
inferiorly in anterior chamber.
a) What is your probable diagnosis?
b) What other signs will you look for on S/L exam?
c) What is the treatment plan?

ANS:
a) ACUTE IRIDOCYCLITIS
b) SIGNS TO LOOK FOR ON SLIT LAMP EXAMINATION:
1. Limbus examination for circumcorneal congestion
2. Cornea for Keratic Precipitates (pathognomonic of this condition)
3. Anterior chamber for Aqueous flare, Aqueous Cells, Fibrinous Exudates
4. Anterior Vitreous for Inflammatory Cells
5. Iris is muddy
c) TREATMENT PLAN:
1. Rest to the eye is given by dilatation of the pupil with 1% atropine. (paralysis ciliary body
and iris, diminishes hyperaemia and prevents formation of posterior syneciae)
2. Dark glasses or an eyeshade may also be used to avoid glare, discomfort and lacrimation
especially in sunlight.
3. Heat application improves blood circulation and relieves pain.
4. Control of acute phase of the inflammation with corticosteroids.
i. Topical Eyedrops and eye ointment, (dexamethasone)
ii. Subconjunctival injection, (betamethasone)
iii. Periocular injection of depot steroids (triamcinolone)
iv. Systemic steroids full course with maintenance tapering doses, (prednisolone)

KMU Past Papers Solved SEQs


Uvea, Lens and Vitreous 37

5. Analgesics and anti-inflammatory relieve pain and discomfort, e.g. aspirin, ibuprofen, etc.
6. Modern broad-spectrum antibiotics which cross the blood-aqueous barrier are given in
cases of Infections.
7. Non-steroidal Anti-inflammatory (NSAIDs) and Cytotoxic Drugs (cyclosporine)
8. Specific treatment of the underlying disease should be added if the etiology is identified e.g.
Reiter’s syndrome, syphilis, tuberculosis etc.

Q-10. A young man with a dark complexion and eyes, presents at the OPD with a sore Red eye,
decreased vision, and photophobia. For the last 3 days it has got progressively worse. He has also
been suffering from a chronic backache for over a year. On examination: RVA 6/36 Unaided
Circum-corneal injection Cells +4-+4 Flare +4, Posterior Synechiae LVA 6/9 Cells +
a) What is the likely diagnosis and its likely underlying cause?
b) What tests would you do to confirm the diagnosis, and its underlying cause?
c) What topical and systemic treatments are available to treat this disease?
d) What are the potential consequences of both ocular and systemic conditions?

ANS:
a) ACUTE IRIDOCYCYLITIS,
Cause is ankylosing spondylitis or sacroililits
b) INVESTIGATIONS:
1. Hematological investigations:
i) Total and differential white blood cell count (TLC and DLC) is done to have a general
information about inflammatory response of the body.
ii) ESR to ascertain existence of any chronic inflammatory condition.
iii) Blood sugar levels to rule out diabetes mellitus.
iv) Blood uric acid in patients suspected of having gout.
v) Serological tests for syphilis, toxoplasmosis, and histoplasmosis.
vi) Tests for antinuclear antibodies, rheumatoid factor, C-reactive proteins
2. Urine examination is done for WBC, pus cells, RBC and culture to rule out urinary tract
infections.
3. Stool examination is done for cyst and ova to rule out parasitic infestations.
4. Radiological investigations include X-rays of chest, paranasal sinuses, sacroiliac joints and
lumbar spine.
5. Skin test: These include tuberculin test, Kveim’s test (sarcoidosis) and toxoplasmin test.
c) TREATMENT:
1. Rest to the eye is given by dilatation of the pupil with 1% atropine.
2. Dark glasses or an eyeshade may also be used to avoid glare, discomfort and lacrimation
especially in sunlight.
3. Heat application improves blood circulation and relieves pain.
4. Control of acute phase of the inflammation with corticosteroids.
i) Topical Eyedrops and eye ointment, (dexamethasone)
ii) Subconjunctival injection, (betamethasone)
iii) Periocular injection of depot steroids (triamcinolone)
iv) Systemic steroids full course with maintenance tapering doses, (prednisolone)
5. Analgesics and anti-inflammatory relieve pain and discomfort, e.g. aspirin, ibuprofen, etc.
6. Modern broad-spectrum antibiotics which cross the blood-aqueous barrier are given in
cases of Infections.
7. Non-steroidal Anti-inflammatory (NSAIDs) and Cytotoxic Drugs (cyclosporine)
8. Specific treatment of the underlying disease should be added if the etiology is identified
e.g. Reiter’s syndrome, syphilis, tuberculosis etc.
d) COMPLICATIONS:
1. Complicated cataract

KMU Past Papers Solved SEQs


Uvea, Lens and Vitreous 38

2. Secondary glaucoma
3. Cyclitic membrane
4. Choroiditis
5. Retinal complications—These include cystoid macular oedema, macular degeneration,
exudative retinal detachment and secondary periphlebitis retinae.
6. Band-shaped keratopathy
7. Phthisis bulbi—The eye becomes soft and shrinks due to hypotony.

Q-11. A 20 years girl complains of dimness of vision in the left eye for the last six days. They also give
history of trauma to the right eye six months previously.
a) Give most probable diagnosis.
b) What ocular signs will make the diagnosis?
c) Give possible treatment

ANS:
a) SYMPATHETIC ENDOPHTHALMITIS
b) OCULAR SIGNS:

1.
Eye is frequently red and irritable loss of accommodation.
2.
Granulomatous anterior uveitis
3.
Multifocal choroidal infiltrates develop in the midperiphery
4.
Dalen–fuchs nodules.
5.
Exudative retinal detachment,
6.
Vasculitis
7.
Optic disc swelling
8.
Residual chorioretinal scarring may confer a ‘sunset glow’
c) TREATMENT:
1. Steroids:
i) High dose oral prednisolone is given for several months, and gradually tapered
according to response.
ii) Intravenous methylprednisolone may be used in some cases.
iii) Supplementary topical steroids and cycloplegics may be given to target anterior
uveitis, and peri- and intraocular steroids, including slow-release intravitreal
implants, may facilitate reduced systemic treatment.
2. Immunosuppressive Such as azathioprine, cyclosporine and methotrexate can be used
in resistant cases or as steroid-sparing agents.
Biological blockers (e.g. infliximab, adalimumab) may be considered
3. Enucleation: of a severely injured eye in the first week. It may be considered for an
injured eye with a hopeless visual prognosis. Evisceration has conventionally been
viewed as inadequate, though recent evidence has raised the possibility of a protective
effect provided all uveal tissue is removed.

KMU Past Papers Solved SEQs


Uvea, Lens and Vitreous 39

Q-12. A healthy young man of 25 years presents with history of blurry vision and photophobia in
his right eye. On examination there are cells and keratic precipitates in the anterior chamber.
a) What is the most likely diagnosis?
b) What signs will make your diagnosis certain?
c) What are the complications of this disease?

ANS:
a) ANTERIOR UVEITIS
b) DIAGNOSIS CERTAIN BY:
1. Hematological Investigations:
i) TLC and DLC
ii) ESR
iii) Blood Glucose
iv) Blood Uric Acid Levels
v) Serological Tests for Syphilis, Toxoplasmosis and Histoplasmosis
vi) Test for anti-nuclear antibodies, rheumatoid factor, LE cells, C – reactive proteins,
etc
2. Urine Examination:
i) WBC and RBC
ii) Pus Cells
iii) Cultures to rule out UTI’s
3. Stool Examination:
i) Parasitic Infections
4. Radiological Examination:
i) X – Ray Chest, Paranasal Sinuses, Sacroiliac Joints and Lumbar Spine
5. Skin Test:
i) Tuberculin Test
ii) Kveim’s Test (Sarcoidosis)
iii) Toxoplasim test
c) COMPLICATIONS:
1. Complicated Cataract
2. Secondary Glaucoma
3. Cyclitic Membrane
4. Choroiditis
5. Retinal Complications – Cystic Macular Edema, Macular Degeneration, Retinal
Detachment
6. Band – Shaped Keratopathy
7. Phthisis Bulbi

Q-13. An old man presented with a history of gradual decrease in the left eye for the past 6
months. There is no pain or any other symptoms. On distant direct ophthalmoscopy there is
dull red reflex in the center. All other structures are normal
a) What is the most common diagnosis?
b) How would you assess the retinal functions?
c) What are the methods of treatment and which is the most preferred one?

ANS:
a) CATARACT

KMU Past Papers Solved SEQs


Uvea, Lens and Vitreous 40

b) ACCESS RETINAL FUNCTION:


1. Projection of Light
2. Macular Functions Test:
ii) Two Point Discrimination Test
iii) Maddox Rod Test
iv) Entoptic View of Retina
v) Foveal Electroretinogram (ERG)
vi) Ultrasonic Investigation by B – Scan
vii) Laser Interference Fringes
viii) Photo Stress Test
c) METHODS OF TREATMENT:
1. Intracapsular Cataract Extraction
2. Extracapsular Cataract Extraction
3. Phacoemulsification
Preferred Method: Extracapsular Cataract Extraction *Jogi Reference*

KMU Past Papers Solved SEQs


Retina and Optic nerve 41

Q-1. A 40 years old female presents with diplopia. She also gives history of headache and
vomiting for the last two months. On examination her Visual Acuity is 6/9 both eyes. There is
Esotropia with limited abduction both eyes. Both optic discs are swollen and there is no
venous pulsation.
a) What is the most likely diagnosis?
b) Briefly discuss three differential diagnoses?
c) Briefly describe your management plan for this patient?

ANS:
a) PAPILLOEDEMA
b) DIFFERENTIAL DIAGNOSIS:
1. Optic Neuritis
2. Anterior Optic Neuropathy (ischemic or compressive)
3. Pseudopapilloedema
4. Mitochondrial optic neuropathies
i) Leber hereditary optic neuropathy
ii) Methanol poisoning
5. Intraocular disease
i) Central retinal vein occlusion
ii) Uveitis
iii) Posterior scleritis
6. Malignant hypertension
7. Meningitis
8. Astigmatism
9. Toxemia of pregnancy
c) MANAGEMENT:
Papilledema indicates raised intracranial pressure and a neurological emergency.
1. Treat the underlying cause of papilloedema.
2. Surgical decompression should be done early, i.e. before the visual field changes occur.
3. There is rapid recovery of vision after the decompression usually.

Q-2. A 30 years old female with high myopia gives history of sudden onset of flashes, floaters
and a shadow over the inferior field of vision.
a) What is the most likely diagnosis?
b) How will you asses the eye? (Methods of examination)
c) List the treatment options available?

ANS:
a) RETINAL DETACHMENT
b) METHODS OF EXAMINATION:
1. Plane Mirror Examination – Defective or No red glow
2. Fundus Examination – Direct and Indirect Ophthalmoscopy
i) Detached Retina looks Greyish – white and raised above surface
ii) Retinal Vessels are dark with no central light reflex
iii) Detached retina is thrown into multiple folds which oscillate with the movement of the eye
iv) Degeneration, pigmentation and hemorrhage of retina may be seen
3. Visual Fields – Scotomas at area corresponding to retinal detachment
4. Electroretinography (ERG) – subnormal or absent

KMU Past Papers Solved SEQs


Retina and Optic nerve 42

5. Ultrasonography – confirms diagnosis in cases retina cannot be visualized


c) TREATMENT OPTIONS:
1. To Seal Retinal Breaks:
i) Photocoagulation
ii) Cryosurgery
2. To Approximate the Sclera, Choroid and Detached Retina:
i) Scleral Buckling
ii) Drainage of Subretinal Fluid (SRF)
iii) Pars Plana Vitrectomy
iv) Pneumoretinopexy

Q-3. A two years old child is brought to eye OPD with white papillary reflex (leukocoria) in his
right eye.
a) What are the differential diagnoses?
b) How will you reach at the diagnosis?

ANS:
a) DIFFERENTIAL DIAGNOSIS:
1. Retinoblastoma
2. Congenital Cataract
3. Persistant Hyperplastic Vitreous
4. Exudative retinopathy of coats (retinal telangiectasia)
5. Choroidal coloboma
6. Retinal dysplasia.
7. Retrolental fibroplasia
8. Toxocara infection
b) APPROACH TO DIAGNOSIS:
2. Red reflex testing with a direct ophthalmoscope is a simple screening test for leukocoria that
is easily employed in the community.
3. Examination under anaesthesia includes the following:
i) General examination for congenital abnormalities of the face and hands.
ii) Tonometry.
iii) Measurement of the corneal diameter.
iv) Anterior chamber examination with a hand-held slit lamp.
v) Cycloplegic refraction.
4. Enzyme levels lactate dehydrogenase and phosphoglucose isomerase enzyme levels are
raised in aqueous humor.
5. Ultrasound is used mainly to assess tumor size. It also detects calcification within the tumor
and is helpful in the exclusion of simulating lesions such as Coats disease.
6. Wide-field photography is useful for both surveying and documentation, and offers
particular advantages in the management of retinoblastoma.
7. Plain X-rays may be used to detect calcification in resource-poor regions.
8. CT also detects calcification but entails a significant dose of radiation so is avoided by many
practitioners.
9. MRI does not detect calcification but is useful for optic nerve evaluation, detection of
extraocular extension and pinealoblastoma.
10. Systemic assessment includes physical examination and MRI scans of the orbit and skull as a
minimum in high-risk cases. If these indicate the presence of metastatic disease then bone
scans, bone marrow aspiration and lumbar puncture are also performed.

KMU Past Papers Solved SEQs


Retina and Optic nerve 43

11. Genetic studies on tumour tissue and blood samples from the patient and relatives

Q-4. The parents of 1 year old child think that the child cannot see for the last few months.
They have also noticed that his pupils are progressively getting white in both eyes.
a) What are the causes of a white pupil in a child?
b) What should an ophthalmologist examine in this child under anesthesia?
c) List 2 important imaging studies used in the management of this child
d) Why is it important to examine the fundus of child with Squint?

ANS:
a) WHITE PUPIL CAUSES:
1) Retinoblastoma
2) Congenital Cataract
3) Persistant Hyperplastic Vitreous
4) Exudative retinopathy of coats (retinal telangiectasia)
5) Choroidal coloboma
6) Retinal dysplasia.
7) Retrolental fibroplasia
8) Toxocara infection
b) EXAMINATION UNDER ANESTHESIA:
Fundus by indirect ophthalmoscopy and sclera indentation under general anesthesia
c) IMAGING STUDIES:
1. X – Ray Orbit
2. MRI/CT – Scan
d) TO EXCLUDE ANY RETINAL PATHOLOGY

Q-5. A 25 years female patient presents with history of headache which is more severe in the
morning. Associated with diplopia, projectile vomiting and transient loss of consciousness for
the last 4 days.
a) What is the most likely diagnosis?
b) What single clinical examination is required to help in the diagnosis?
c) What radiological investigation will confirm the final diagnosis?

ANS:
a) INTRACRANIAL HYPERTENSION PAPILLO-EDEMA
b) FUNDOSCOPY for fundus examination
c) RADIOLOGICAL INVESTIGATIOS:
1. B-scan ultrasonography can be used to aid in distinguishing between papilloedema and other
causes of a swollen or apparently swollen optic disc (pseudopapilloedema)
2. MRI to exclude a space-occupying lesion and enlarged ventricles.
3. MRI can also be used to measure optic nerve sheath diameter ONSD (average normal
diameter approximately 5.5 mm ± 1 mm on MRI).
4. Vascular imaging may be performed, such as venography to rule out cerebral venous sinus
thrombosis.

KMU Past Papers Solved SEQs


Retina and Optic nerve 44

Q-6. A 24 year old presents with a history of headache and vomiting from the past 3 weeks. He
complained of having attacks of blacking out or graying out of vision lasting 10-15 Seconds often
precipitated by sudden change of posture. Fundus examination revealed the following findings in
the both eyes. The optic discs were hyperemic and elevated. The margins of the discs being
blurred. Multiple hemorrhages and exudates were seen around the disc. Venous pulsations were
absent
a) What condition does the above given ocular findings most likely suggest?
b) Give 3 possible differentials.
c) What is the investigation of choice?

ANS:
a) BILATERAL SWOLLEN DISC DUE TO BENIGN INTRA CRANIAL HYPERTENSION
PAPILLOEDEMA
b) DIFFERENTIAL DIAGNOSIS:
1. Optic Neuritis
2. Anterior Optic Neuropathy (ischemic or compressive)
3. Pseudo-papilloedema
4. Mitochondrial optic neuropathies
i) Leber hereditary optic neuropathy
ii) Methanol poisoning
5. Intraocular disease
i) Central retinal vein occlusion
ii) Uveitis
iii) Posterior scleritis
6. Malignant hypertension
7. Meningitis
8. Astigmatism
9. Toxemia of pregnancy
c) INVESTIGATION OF CHOICE
1. For eye:
i) B-scan ultrasonography
ii) Ocular coherence tomography (OCT)
2. Systemic:
i) MRI and MRI Angiography
ii) CT- Scan
iii) Venography

Q-7. A 65 year Old obese female with valvular heart disease. Comes with complain if sudden
painless visual loss. She has a past history of repeated transient episodes of visual loss, and
that it sometimes occurs in one eye and sometimes in the other. On examination she had a
poor vision and the direct pupillary reflex was absent. funduscopy showed thinning of the
arteries, the veins being normal. a cherry red spot was also seen in the macular region.
a) What is the most probable diagnosis?
b) What is this phenomenon called when there are transient episodes if unilateral blindness
which resolves itself after a while?
c) What is the most common etiology for this condition?

ANS:

KMU Past Papers Solved SEQs


Retina and Optic nerve 45

a) DIAGNOSIS:
Central Retinal Artery Occlusion, secondary to embolic phenomenon form the vegetation of
abnormal heart valves.
b) AMAUROSIS FUGAX
c) ETIOLOGY:
1. Thrombosis due to atherosclerosis at the level of lamina cribrosa (most common)
2. Embolism from atherosclerotic plaque
3. Angiospasm (retinal migraine)
4. Periarteritis (SLE, polyarteritis nodosa)
5. Temporal arteritis
6. Buerger’s disease.
7. Raised intraocular pressure after retinal surgery

Q-8. A 65 year old presents with sudden profound decrease of vision in the left eye, he presents
to the A&E department within 30 minutes of loss of vision where you are the Medical Officer
the visual acuity is 6/6 in the Right eye and Hand Movement in the left eye. He has marked
Left Relative Afferent Pupillary Defect, Anterior Segments and Intra Ocular Pressures are
normal. There are NO medical Opacities. The Left Retina is pale with a cheery red spot. The
Optic disc is also pale and both the arteries and veins show segmentation. He smokes 20
cigarettes a day and has a BMI of 30.
a) What is the diagnosis?
b) What emergency treatment you can carry out?
c) What Cardiovascular examination would you order?
d) What advice would you give to the patient to prevent the same? In other eye.

ANS:
a) CENTRAL RETINAL ARTERY OCCLUSION
b) EMERGENCY TREATMENT:
1. Adoption of a supine posture might improve ocular perfusion and should always be
implemented.
2. Ocular massage to mechanically collapse the arterial lumen and cause prompt changes in
arterial flow, improving perfusion and potentially dislodging an embolus or thrombus.
3. Anterior chamber paracentesis to withdraw 0.1–0.2 ml of aqueous to produces
immediate hypotony causing vasodilatation.
4. Topical apraclonidine, timolol and intravenous acetazolamide to achieve a more
sustained lowering of intraocular pressure
5. Hyperosmotic agents. Mannitol or glycerol have been used for their possibly more rapid
IOP-lowering effect as well as increased intravascular volume.
6. Sublingual isosorbide dinitrate to induce vasodilatation.
7. ‘Rebreathing’ into a paper bag in order to elevate blood carbon dioxide and respiratory
acidosis has been advocated, as this may promote vasodilatation.
8. Breathing a high oxygen (95%) and carbon dioxide (5%) mixture, ‘carbogen’, has
been advocated for a possible dual effect of retarding ischaemia and vasodilatation.
9. Transluminal Nd:YAG laser embolysis/embolectomy has been advocated for BRAO or
CRAO in which an occluding embolus is visible. Embolectomy has been said to occur if the
embolus is ejected into the vitreous via a hole in the arteriole.
10. Thrombolysis. By giving streptokinase I/V to dissolve the emboli.
11. Retrobulbar tolazoline injection to decrease the retrobulbar resistance to blood flow.
c) CARDIOVASCULAR EXAMINATION:
1. ECG

KMU Past Papers Solved SEQs


Retina and Optic nerve 46

2. Echocardiography
3. Lipid profile: (Serum cholesterol, Serum LDL, Triglyceride, Serum HDL)
4. Carotid duplex scanning
d) ADVICE:
1. Avoid fatty diet
2. Exercise daily
3. Quiet smoking

Q-9. A 65-year-Old man complaints of sudden loss of vision in his right eye. he suffered no pain
a) What are the differential diagnosis?
b) How will you establish your diagnosis?

ANS:
a) HOW TO ESTABLISH DIAGNOSIS:
1. History and examination focusing on above mentioned causes,
2. Blood pressure
3. Blood sugar
4. Fundus examination:
i) Cherry red spot, Cattle tracking sign (CRAO)
ii) Flame Shape Hemorrhage, Cotton Wool Spots, Tomato Splash Appearance (CRVO)
iii) No Red Glow on Fundoscopy (Vitreous Hemorrhage)
iv) Retinal breaks in Retinal Detachment
5. Fundus Fluorescein Angiography (FFA)
6. Indocynanine Green Angiography
7. Optical Coherence Tomography
8. B- scan
9. MRI brain

Q-10. A 24 years old male presented with a history of headache and vomiting from the last 3
weeks, he complained of having attack of blacking out or greying out Of vision lasting 10 to 15
seconds often precipitated by sudden Change Of posture fundus examination revealed the
following finding in both eyes. Optic disc and elevated. The margin of the disc being blurred
and multiple hemorrhages and exudates were seen around the disc venous pulsation were
absent.
a) What condition does the above given ocular findings most likely suggest?
b) Give three possible differentials.
c) What is the investigation of choice?

ANS:
a) BILATERAL SWOLLEN DISCS DUE TO BENIGN INTRACRANIAL HTN (PAPILLOEDEMA)
b) DIFFERENTIAL DIAGNOSIS:
1. Optic Neuritis
2. Anterior Optic Neuropathy (ischemic or compressive)
3. Pseudopapilloedema
4. Mitochondrial optic neuropathies
i) Leber hereditary optic neuropathy
ii) Methanol poisoning

KMU Past Papers Solved SEQs


Retina and Optic nerve 47

5. Intraocular disease
i) Central retinal vein occlusion
ii) Uveitis
iii) Posterior scleritis
6. Malignant hypertension
7. Meningitis
8. Astigmatism
9. Toxemia of pregnancy
c) INVESTIGATIONS:
 Systemic investigation
1. Full blood count
2. ESR for the existence of any chronic inflammatory condition
3. Blood sugar levels to rule out diabetes mellitus
4. Lumbar puncture (LP) must not be carried out until imaging has excluded a space-
occupying lesion.
 Imaging:
1. B-scan ultrasonography can be used to aid in distinguishing between papilloedema and
other causes of a swollen or apparently swollen optic disc (pseudopapilloedema)
2. MRI to exclude a space-occupying lesion and enlarged ventricles.
3. MRI can also be used to measure optic nerve sheath diameter ONSD (average normal
diameter approximately 5.5 mm ± 1 mm on MRI).
4. Vascular imaging may be performed, such as venography to rule out cerebral venous
sinus thrombosis.

Q-11. A 60 year Old gentleman present with progressive decrease of vision in both eyes for the
last 2 years. His BMI is 32 and he is getting recurrent skin infections for some years. He has
noticed that he is getting tired after eating meals and feels thirsty most of the times with
frequency of micturition. His visual acuities are 6/18 and 6/9 Right and eyes respectively with
no improvement with pinhole. Pupillary reactions are normal. Intraocular pressures are
within normal limits. Slit lamp examination of Anterior Segment is unremarkable. There are
no medial opacities on distant direct Ophthalmoscopy, Direct Ophthalmoscopy reveals
bilateral macular edema with hard exudates and micro aneurysms scattered in the both
Retina. His Blood Pressure is 140/90mmHg, His urine is positive for Glucose with traces of
proteins. His fasting lipid profile show raised cholesterol.
a) What is the likely cause of macular edema?
b) What blood tests would your order to determine the cause of Glucose in urine?
c) What urine tests would you do to further evaluate proteins in urine?
d) What imaging tests would you perform to confirm Macular Edema?
e) What is the systemic and ocular therapeutic management of Macular edema in this case?

ANS:
a) DIABETIC RETINOPATHY
b) BLOOD GLUCOSE AND HBA1C
c) URINE DIPSTICK
d) OPTIC COHERANCE TOMOGRAPHY
e) TREATMENT:
 Ocular:
1. Intravitreal corticosteroids, Triamcinolone acetonide, Dexamethasone
2. Intravitreal anti-VEGF agents

KMU Past Papers Solved SEQs


Retina and Optic nerve 48

3. Laser photocoagulation
4. Pars Plana Vitrectomy
 Systemic:
1. Glucose control
2. Diet modification
3. Exercise and weight reduction

Q-12. A 50 years old diabetic woman for the last 8 years complains of decreased vision for the last 3
weeks.
a) How diabetic complications can present in this way?
b) How will you treat such a patient?

ANS:
COMPLICATIONS:
1. Retinopathy.
i. Background retinopathy
ii. Pre-proliferative retinopathy
iii. Proliferative retinopathy
iv. Diabetic macular edema
v. Tractional retinal detachment
2. Accelerated senile cataract.
3. vitreous hemorrhage
4. Neovascular glaucoma (NVG).
TREATMENT:
 General
1. Patient education is critical, regarding the treatment schedules in order to optimize
visual outcomes.
2. Diabetic control should be optimized.
3. Smoking should be discontinued, though this has not been definitively shown to
affect retinopathy.
4. Other risk factors, particularly systemic hypertension (especially type 2 diabetes)
and hyperlipidemia should be controlled in conjunction with the patient’s
diabetologist.
5. Fenofibrate 200 mg daily has been shown to reduce the progression of diabetic
retinopathy in type 2 diabetics and prescription should be considered; the decision is
independent of whether the patient already takes a statin.
6. Other modifiable factors such as anemia and renal failure should be addressed as
necessary.
 Specific:
1. Laser photocoagulation (Focal/Grid) by Argon or Diode Lasers
2. Sub-threshold micro pulse diode laser.
3. Intravitreal anti-VEGF agents
4. Intravitreal triamcinolone
5. Pars plana vitrectomy
6. Cryotherapy

KMU Past Papers Solved SEQs


Retina and Optic nerve 49

Q-13. A 25 years old presented with extreme disturbance of vision at night. On exam he has VA
of 6/6 each eye. His maternal uncle also had the same problem. He has bilateral severe visual
field constriction.
a) What is the most probable diagnosis?
b) Name the three fundus findings which will support your diagnosis.
c) Name the relevant investigations in the case.
d) What are the treatment options?

ANS:
a) RETINITIS PIGMINTOSA
b) FUNDUS FINDING:
1. Jet black spots on retina (spidery outline)
2. Retinal Vessels become extremely attenuated and thread – like
3. Pale wax – like, yellowish Appearance of Optic Disc
c) INVESTIGATIONS:
1. Visual Fields – Annular or Ring Scotoma causing tubular vision
2. Electroretinogam and Electro – oculogram – Subnormal
d) TREATMENT OPTIONS:
1. Genetic Counseling
2. Low Vision Aids
i) Near Vision – Hand and Stand Magnifier
ii) Telescopes – Monocular or binocular telescope of various magnification
iii) Electronic Aids – Closed Circuit Television

Q-14. A 65 years old hypertensive patient presented with sudden decrease in vision in his right
eye for the last 2 days. On examination, his Best Corrected Visual Acuity (BCVA) in his right
eye is 3/60 and left eye is 6/6. Both eyes are pseudophakic. Right fundus shows extensive and
diffusely scattered retinal hemorrhages, cotton wool spots, retinal edema and macular edema
while left eye is normal. Right eye also shows Relative Afferent Pupillary Defect (RAPD).
a) What is your diagnosis?
b) What ocular condition predispose to this conditions?
c) What are the complication?
d) What are the treatment options?

ANS:
a) CENTRAL RETINAL VEIN OCCLUSION (CRVO)
b) OCULAR CONDITIONS PREDISPOSING TO CRVO:
1. Hypertensive Retinopathy
2. Branch Retinal Vein Occlusion
3. Serous Detachment of Retina
4. Ischemic Optic Neuropathy
5. Ocular Nerve Palsy leading to Paralytic Squint
c) COMPLICATIONS:
1. Macular Ischemia
2. Cystoid Macular Edema
3. Rubeosis Iridis which cause Neovascular Glaucoma
d) TREATMENT OPTIONS:

KMU Past Papers Solved SEQs


Retina and Optic nerve 50

 Medical:
1. Intravitreal triamcinolone acetone for macular edema
2. Oral steroids
 Laser:
1. Laser photocoagulation
2. Pan retinal photocoagulation is carried out if there is rubeosis iridis
 Surgical:
1. Optic nerve shethotomy via pars-plana approach for decompressing the central vein
at lamina cribrosa
2. Cryotherapy

Q-15. A 60 years old male presents with sudden loss of vision in his right eye. Fundus findings
include venous dilation, wide spread retinal hemorrhage, cotton wool spots, cystoids macular
edema.
a) What is the differential diagnosis?
b) How will you manage such a case?

ANS:
a) DIFFERENTIAL DIAGNOSIS:
1. Central or Branch Retinal Vein Occlusion
2. Central or Branch Retinal Artery Occlusion
3. Retinal Detachment
b) MANAGEMENT:
 Work for the diagnosis:
1. Visual acuity
2. Fundoscopy Whitish looking fundus with cherry red spot (fovea centralis) in artery
occlusion, dilated tortuous veins in CRV occlusion
3. Fundus flourescein angiography to assess the perfusion of retina
 Treatment:
1. CRV occlusion (Oral steroids, Intravitreal triamcnolone, optic nerve sheathotomy (for
decompressing CRV), Pan-retinal photo-coagulation in all eyes with rubeosis iridis
2. CRA occlusion (Lower IOP by Mannitol, Acetazolamide, paracentesis of anterior chamber)

Q-16. A young myopic female patient of thirty years presents with sudden painless loss of vision
in her left eye. Her vision is 6/6 with glasses in the unaffected eye and CF (counting fingers) in
the affected eye with glasses. She gives history of flashes of light in the affected eye in the
recent past.
a) What is the most likely diagnosis?
b) Keeping this scenario in mind what is the other possible diagnosis?
c) What important clinical examination must be done in this case?

ANS:
a) RETINAL DETACHMENT
b) POSSIBLE DIAGNOSIS:
1. Central Retinal Artery Occlusion
2. Central Retinal Vein Occlusion
3. Optic neuritis: papillitis, retrobulbar neuritis

KMU Past Papers Solved SEQs


Retina and Optic nerve 51

4. Vitreous Hemorrhage
5. Retinal Detachment
6. Anterior ischaemic optic neuropathy (AION)
7. Cortical lesions—Trauma, vascular
8. Central serous chorioretinopathy
9. Dislocation of Lens
c) FUNDOSCOPY (OPHTHALMOSCOPY)

Q-17. A 30-Year-old Caucasian lady working for an NGO is referred to you with gradual decrease
of vision in left eye for last 5 days. She is otherwise fit but latterly has noticed that she feels
numbness of fingers after a hot shower. On Examination of Eye.
Right Eye findings are, VA '6/6, No Afferent Papillary defect, fundus is normal intraocular pressure
is normal color vision is normal
Left eye findings are, VA 6/6 relative afferent papillary defect, Anterior Segment is Normal, Fundus
is Normal, Intra Ocular Pressure is Normal, Color Vision Red/Green defect.
a) What is the likely diagnosis?
b) How would you confirm the diagnosis?
c) What is the treatment of the ocular condition and suspected underlying diseases?
d) What advice would you give to the patient?

ANS:
a) OPTIC NEURITIS, SECONDARY TO MULTIPLE SCLEROSIS
b) INVESTIGATION:
1. Perimetery: Central or Centrocaecal Scotoma is most common
2. Contrast sensitivity using Perri-robson charts
3. Electrophysiological test: Visual evoked potential is abnormal (conduction delay and a
reduction in amplitude)
4. Optical Coherence Tomography (OCT)
5. MRI brain for paraventricular plaques in multiple sclerosis
6. Blood Complete picture and ESR
7. Serological tests
8. Lumbar puncture shows oligo clonal bands on protein electrophoresis of cerebrospinal
fluid
c) TREATMENT:
 For Ocular Condition:
Corticosteroids therapy for shorten the period of visual loss
 For Multiple Sclerosis:
1. Steroid regimen: Intravenous methylprednisolone sodium succinate
2. Immunomodulatory treatment (IMT): which include interferon beta (IM) teriflunomide
and glatiramer.
3. Vitamin B1 B6 and B12
d) ADVICE
Ensure the patient to complete his medication tell her that visual acuity and color vision usually
recover in months

KMU Past Papers Solved SEQs


Orbit
Retinaand
andInjury
Optictonerve
Eye 52

Q-18. A 6 years old child is presented to you with a white pupillary reflex. What could by the
differential diagnosis and how you will establish a diagnosis?

ANS:
a) DIFFERENTIAL DIAGNOSIS:
1. Retinoblastoma
2. Congenital Cataract
3. Persistant Hyperplastic Vitreous
4. Exudative retinopathy of coats (retinal telangiectasia)
5. Choroidal coloboma
6. Retinal dysplasia.
7. Retrolental fibroplasia
8. Toxocara infection
b) TO ESTABLISH DIAGNOSIS:
1. History:
i) History of difficult delivery and prematurity,
ii) Family history of retinoblastoma and congenital cataract in other siblings
2. Examination: For any discharge, proptosis
3. Investigations:
i) Indirect Ophthalmoscopy
ii) Slit Lamp Exam
4. Imaging:
i) B-scan
ii) X ray
iii) CT- scan
iv) MRI
5. Serological: TORCH serology, serum calcium
6. Urine for reducing substances

Q-19. A 35 years old female presented with rapid loss of vision over two days in her right eye.
Ocular examination shows relative afferent papillary defect (RAPD) and Swollen Optic disc.
a) What are the differential diagnoses?
b) How will you investigate such a case?

ANS:
a) DIFFERENTIAL DIAGNOSIS:
1. Ischemic Optic Neuropathy
2. Lebers Optic Neuropathy
3. Toxic or Metabolic Optic Neuropathy
b) INVESTIGATION:
1. Fundus Examination:
i) Swollen disc with or without peripapillary flame – shaped hemorrhages
2. Visual Fields:
ii) Central Scotoma
iii) Concentric Loss
3. Visual Evoke Potential:
iv) Abnormal

KMU Past Papers Solved SEQs


Orbit and Injury to Eye 53

Q-1. You are sitting in your clinic and an 8 years old child is brought to you with a trauma to his
right eye with a stone. He complains sudden drop of vision in the affected eye. On examination
with torch you find blood collection behind the cornea with a fluid level.
a) What is your diagnosis?
b) Name single most important clinical test?
c) What are steps for management?

ANS:
a) HYPHAEMA
b) SLIT LAMP EXAMINATION
c) MANAGEMENT:
1. Advise patient for Bed Rest
2. Topical Steroids
3. Topical Cycloplegics
4. Topical/Systemic Intraocular Pressure Lowering Medicines
5. Systemic Antifibrinolytic Agents like Tranexamic Acid
6. Anterior Chamber Wash
7. Paracentesis
8. Trabeculectomy

Q-2. A 35-year-Old mechanic was hammering metal when he felt something went into his Right
Eye. He presents to the Casualty Department for a checkup. His Visual acuities are 6/6 in both
eyes with NO Pupillary deficit. Examination of the anterior segments is normal apart from a
sub conjunctival Hemorrhage 5mm from Limbus at 9-degree position in his Right eye.
a) What further examinations is necessary in this case
b) What imaging studies are mandatory in this patient to rule out intra Ocular Foreign Body?
c) What is the role of Magnetic Resonance Imaging in this patient?
d) What initial treatment would you carry out as Medical Officer in Casualty Department?
e) Most Casualty units have ophthalmoscope but no Slit lamp. Enumerate signs of
Endophthalmithis which can be elicited without the use of slit Lamp?

Ans;
a) LOCALIZATION OF FOREIGN BODY VIA
1. Direct visualization by Magnifying Glass
2. Oblique illumination with a loupe
3. Goniolense,
4. Slit lamp along with Direct and Indirect ophthalmoscopy
b) IMAGING STUDIES REQUIRED:
1. X-ray Orbit
2. Ultrasonography A and B scan
3. CT scan
c) ROLE OF MAGNETIC IMAGING:
MRI contra-indicated in suspected metallic foreign body, because strong magnetic field of MRI
machine will put traction on metallic body, causing further damage
d) INITIAL TREATMENT
1. Advice to not rub the eyes, as the foreign body may penetrate in the deeper tissues.
2. Wash the eye with Normal saline, or plane water,
3. Give analgesic to for pain,

KMU Past Papers Solved SEQs


Orbit and Injury to Eye 54

4. re-assure the patient and refer to Ophthalmologist


e) SIGNS ELICITED WITHOUT SLIT LAMP:
1. Conjunctival congestion and chemosis
2. Hazy cornea
3. Fibrinous exudates and Hypopion in anterior chamber
4. Loss of Red reflex on Ophthalmoscopy
5. On digital (finger) examination Raised IOP (initially)

Q-3. A 16 years old female presents with rapid onset of left proptosis, lid swelling, fever, pain
visual and visual impairment for the last two days. She gives history of penetrating injury of
left upper lid three days back.
a) What is the most likely diagnosis?
b) How will you manage this patient?
c) What are the complications of this condition?

ANS:
a) ORBITAL CELLULITIS
b) MANAGEMENT:
 Medical treatment:
1. Hospital admission is mandatory, with urgent otolaryngological assessment and
frequent ophthalmic review.
2. Delineation of the extent of erythema on the skin using a surgical marker may help in
judging progress.
3. Hot compress relieves pain and prevents stasis.

4. Antibiotics are given intravenously, ceftazidime is a typical choice, supplemented by oral


metronidazole to cover anaerobes. Intravenous antibiotics should be continued until the
patient has been apyrexial for 4 days, followed by 1–3 weeks of oral treatment.
5. Analgesics and anti-inflammatory drugs are helpful in controlling pain and fever.
6. Monitoring of optic nerve function is performed at least every 4 hours initially by
testing Visual Acquity, colour vision, light brightness appreciation and pupillary reactions.
Deterioration should prompt the consideration of surgical intervention.
 Surgery treatment:
1. Drainage of an orbital abscess should be considered at an early stage.
2. Drainage of infected sinuses should be considered if there is a lack of response to
antibiotics.
3. Biopsy of inflammatory tissue may be performed for an atypical clinical picture.
4. Severe optic nerve compression may warrant an emergency canthotomy/cantholysis.
c) COMPLICATION:
 Ocular complications include
1. Optic neuropathy
2. Exposure keratopathy,
3. Raised IOP
4. Endophthalmitis and
5. Occlusion of the central retinal artery or vein
6. Subperiosteal abscess, (medial orbital wall)
 Intracranial complications (uncommon but extremely serious), includes
1. Meningitis,
2. Brain abscess and
3. Cavernous sinus thrombosis.

KMU Past Papers Solved SEQs


Orbit and Injury to Eye 55

Q-4. A 24 Years Old male is brought to you with severe pain watering and photophobia in both
eyes, He give a history of exposure to electric welding arc about 10 hours back
a) How will you manage to examine such painful eye?
b) What is the most probable diagnosis and how will you confirm it?
c) What is the treatment?

ANS:
a) EXAMINATION OF PAINFUL EYE:
3. Put Local anesthetic drops e.g. proparacaine (alcaine drops) in eye
4. After two minutes examine the conjunctiva and cornea with Slit lamp
b) DIAGNOSIS:
Corneal epithelial erosion or burns due to ultra-violet light of electric welding arc.
To confirm it, after anesthetizing the eye, stain the cornea with 2% Fluorescein dye and then
examine under slit lamp with blue light.
The corneal erosions or ulcerations will take the stain.
This is pathognomonic sign for diagnosis of corneal ulceration.
c) TREATMENT:
1. Decongestants e.g. sodium cromoglycate
2. Lubricants, artificial tears e.g. polyvinyl alcohol
3. Systemic analgesics
4. Protective glasses and safety measures.
5. Counseling

Q-5. A 7 years old child presented with unilateral proptosis, pain fever, decreased ocular
mobility, erythema and edema of the eyelids.
a) What is the diagnosis?
b) What are the differential diagnosis?
c) What investigation will you do to evaluate such case?
d) How will you manage the patient?
e) What are the complications of the condition?

ANS:
a) ORBITAL CELLULITIS
b) DIFFERENTIAL DIAGNOSIS:
1) Inflammatory lesions:
i) Orbital cellulitis
ii) Abscess
2) Vascular disturbances:
i) Hemorrhage
ii) Varicose orbital veins
iii) Hemangioma
3) Cysts and tumors:
i) Dermoid cyst
ii) Osteoma
iii) Lymphoma
iv) Lymphosarcoma
v) Glioma

KMU Past Papers Solved SEQs


Orbit and Injury to Eye 56

vi) Meningioma of optic nerve


vii) Retinoblastoma and
viii) Metastatic deposits in orbit.
4) Systemic diseases:
i) Leukemias
ii) Endocrine disturbances such as Graves’ disease
c) INVESTIGATIONS:
 Laboratory investigations:
1. Blood CP and ESR
2. Blood cultures
3. Culture of nasal discharge
4. Lumbar puncture if meningeal or cerebral signs present.
 Imaging:
1. High resolution CT-scan of orbit, sinuses and brain to confirm diagnosis and exclude
sub periosteal or intracranial abscess.
2. MRI
3. Ultrasonography (B-Scan)
a) MANAGEMENT:
 Medical treatment:
1. Hospital admission is mandatory, with urgent otolaryngological assessment and
frequent ophthalmic review.
2. Delineation of the extent of erythema on the skin using a surgical marker may help in
judging progress.
3. Hot compress relieves pain and prevents stasis.

4. Antibiotics are given intravenously, ceftazidime is a typical choice, supplemented by oral


metronidazole to cover anaerobes. Intravenous antibiotics should be continued until the
patient has been apyrexial for 4 days, followed by 1–3 weeks of oral treatment.
5. Analgesics and anti-inflammatory drugs are helpful in controlling pain and fever.
6. Monitoring of optic nerve function is performed at least every 4 hours initially by
testing Visual Acquity, colour vision, light brightness appreciation and pupillary reactions.
Deterioration should prompt the consideration of surgical intervention.
 Surgery treatment:
1. Drainage of an orbital abscess should be considered at an early stage.
2. Drainage of infected sinuses should be considered if there is a lack of response to
antibiotics.
3. Biopsy of inflammatory tissue may be performed for an atypical clinical picture.
4. Severe optic nerve compression may warrant an emergency canthotomy/cantholysis.
b) COMPLICATIONS:
 Ocular complications include
1. Optic neuropathy
2. Exposure keratopathy,
3. Raised IOP
4. Endophthalmitis and
5. Occlusion of the central retinal artery or vein
6. Subperiosteal abscess, (medial orbital wall)
 Intracranial complications (uncommon but extremely serious), includes
1. Meningitis,
2. Brain abscess and
3. Cavernous sinus thrombosis.

KMU Past Papers Solved SEQs


Orbit and Injury to Eye 57

Q-6. A 20 years old patient comes to the emergency with a history of trauma with fist on the
right eye. On examination he is found to have decreased visual acuity in the right eye and
blood in the anterior chamber
a) What is the condition called?
b) How would you treat such a case?

ANS:
a) HYPHEMA
b) TREATMENT:
 General
1. Strict bed rest is probably unnecessary, patient should remain in a sitting or semi-upright
posture, including during sleep.
2. A protective eye shield should be worn. (patching)
3. Hospital admission may be required for a large hyphaema.
4. Any current anticoagulant medication should be discontinued after liaison with a general
physician to assess the risk;
5. NSAIDs should not be used for analgesia
 Medical
1. Atropine is recommended by some authorities to achieve constant mydriasis and reduce the
chance of secondary haemorrhage, but clear evidence is lacking.
2. Antifibrinolysis with systemic aminocaproic acid (ACA) or tranexamic acid or with topical
ACA may be considered under higher-risk circumstances such as recurrent bleeding.
3. Topical steroids should be used since they reduce inflammation and possibly the risk of
secondary haemorrhage.
4. A beta-blocker and/or a topical or systemic CAI (carbonic anhydrase inhibitor) is
administered, depending on the IOP.
5. Miotics should also be avoided as they may increase pupillary block and disrupt the blood–
aqueous barrier.
 Laser photocoagulation of angle bleeding points via a gonioprism.
 Surgical
evacuation of blood is required, if a total hyphaema or persistently intolerable IOP lasts for
more than 5 days surgery should be considered;
1. To reduce the risk of permanent corneal staining (rare)
2. To reduce the risk of optic atrophy,
3. To prevent the development of peripheral anterior synechiae and
4. To prevent chronic secondary glaucoma.
The surgical options are;
1. Paracentesis of anterior chamber to wash out blood
2. Trabeculectomy to drain the blood and control the intraocular pressure

 Advise: The patient should be advised to avoid any activity with a risk of even minor eye
trauma for several weeks; symptoms of a rebleed should prompt immediate review.

Q-7. A 10 years old boy presents with unilateral proptosis.


a) What is differential diagnosis of it?
b) How will you evaluate such a case?

ANS:
a) DIFFERENTIAL DIAGNOSIS:

KMU Past Papers Solved SEQs


Orbit and Injury to Eye 58

1. Inflammatory lesions:
i) Orbital cellulitis
ii) Abscess
2. Vascular disturbances:
i) Hemorrhage
ii) Varicose orbital veins
iii) Hemangioma
3. Cysts and tumors:
i) Dermoid cyst
ii) Osteoma
iii) Lymphoma
iv) Lymphosarcoma
v) Glioma
vi) Meningioma of optic nerve
vii) Retinoblastoma and
viii) Metastatic deposits in orbit.
4. Systemic diseases:
i) Leukemias
ii) Endocrine disturbances such as Graves’ disease
5. Paralysis of extraocular muscles as in complete ophthalmoplegia.

b) EVALUATION:
 History:
1. Mode of onset
2. Whether sudden, gradual or chronic
3. Presence and duration of pain is important
4. Past history of thyroid dysfunction, orbital trauma, sinus disease and malignancy
5. Family history of retinoblastoma
 Clinical examination: (6 P+)
1. Pain
2. Position of eye ball (axial non axial)
3. Progression
4. Palpation
5. Pulsation
6. Periorbital involvement
7. There is limitations of ocular movements due to oedema, infiltration and fibrosis.
8. Visual acuity may be reduced as a result of exposure keratitis and optic nerve involvement
due to infiltration, pressure by swollen muscle and reduced blood supply.
9. Fundus examination The disc may be normal or show features of optic atrophy, papillitis
or papilloedema.
10. Transillumination and auscultation are done for tumour and pulsating proptosis.
 Laboratory investigations:
1. Blood CP and ESR
2. Random blood sugar
3. Cholesterol
4. Urine examination
5. TFTs (T3,T4, & TSH)
6. ANCA (anti-neutrophil cytoplasmic antibodies)
7. ACE (angiotensin converting enzyme)
 Imaging:
1. X-ray
2. Ultrasonography (B-Scan)

KMU Past Papers Solved SEQs


Orbit and Injury to Eye 59

3. CT-scan
4. MRI
5. Venography
6. Arteriography
 Histopathology studies:
1. FNAC
2. Incisional biopsy
3. Excisional biopsy

Q-8. A22 year Old girl had rhinorrhea and fever for the last three days she complained of pain
in right eye. There is bulging of eyeball the eyeball and her ocular movements are limited the
vision was 6/18 AND 6/6. She is toxic.
a) What is your diagnosis?
b) What is differentiate diagnosis?
c) What relevant investigation would you do in this patient?
d) Give your treatment plan

ANS:
a) ORBITAL CELLULITIS
b) DIFFERENTIAL DIAGNOSIS:
1. Orbital Cellulitis
2. Pre-septal cellulitis
3. Hemangioma
4. Thyroid Opthalmopathy (Grave’s disease)
5. Cyst/tumor (Dermoid cyst, lymphoma, meningioma of the optic nerve, glioma, and
Retinoblastoma)
c) INVESTIGATIONS:
 Laboratory investigations:
1. Blood CP and ESR
2. Blood cultures
3. Culture of nasal discharge
4. Lumbar puncture if meningeal or cerebral signs present.
 Imaging:
1. High resolution CT-scan of orbit, sinuses and brain to confirm diagnosis and exclude
sub periosteal or intracranial abscess.
2. MRI
3. Ultrasonography (B-Scan)

d) MANAGEMENT:
 Medical treatment:
1. Hospital admission is mandatory, with urgent otolaryngological assessment and
frequent ophthalmic review.
2. Delineation of the extent of erythema on the skin using a surgical marker may help in
judging progress.
3. Hot compress relieves pain and prevents stasis.

4. Antibiotics are given intravenously, ceftazidime is a typical choice, supplemented by


oral metronidazole to cover anaerobes. Intravenous antibiotics should be continued
until the patient has been apyrexial for 4 days, followed by 1–3 weeks of oral treatment.
5. Analgesics and anti-inflammatory drugs are helpful in controlling pain and fever.

KMU Past Papers Solved SEQs


Orbit and Injury to Eye 60

6. Monitoring of optic nerve function is performed at least every 4 hours initially by


testing Visual Acquity, colour vision, light brightness appreciation and pupillary
reactions. Deterioration should prompt the consideration of surgical intervention.
 Surgery treatment:
1. Drainage of an orbital abscess should be considered at an early stage.
2. Drainage of infected sinuses should be considered if there is a lack of response to
antibiotics.
3. Biopsy of inflammatory tissue may be performed for an atypical clinical picture.
4. Severe optic nerve compression may warrant an emergency canthotomy/cantholysis.

Q-9. A 40-year-old female had cataract surgery with IOL. On first post-operative day she
complains of moderate pain. On examination her vision is hand movements anterior chamber
shows severe inflammatory response with hazy fundus view.
a) What are possible causes?
b) How will you evaluate this patient?
c) How will you treat this patient?

ANS:
a) POSSIBLE CAUSES:
1. Acute postoperative endophthalmitis
2. Retained Lens Matter causing severe uveitis
3. Vitreous hemorrhage
4. Toxic Anterior Chamber Syndrome
5. Post-operative Iridocyclitis
6. Complicated or prolonged surgery may result in corneal oedema and uveitis.
b) EVALUATION:
1. Check visual acuity
2. Examine both eyes by slit lamp biomicroscopy
3. Intraocular pressure
4. Dilated funduscopy
5. Possible ultrasonography if fundus not well visualized (This will help determine if a
retained intraocular foreign body is present, the density of the vitreitis, and if the retina is
attached or not.)
laboratory study:
1. Gram stain and culture of the aqueous and vitreous
2. Complete blood count with differential - Evaluating for signs of infection, elevated white
count.
3. Blood culture
4. Real-time polymerase chain reaction (RT-PCR) has improved diagnostic results over
traditional culture.
Ocular investigations:
6. Ocular CT and MRI scan
c) TREATMENT:
1. Hospitalize the patient
2. Intravitreal antibiotics are the key to management. Antibiotics commonly used in
combination are ceftazidime, and vancomycin.
3. Subconjuctival Antibiotic Injections (amikacin if penicillin allergic)
4. Topical Antibiotics are of limited benefit and are often used only 4–6 times daily in order
to protect the fresh wounds from contamination.

KMU Past Papers Solved SEQs


Orbit and Injury to Eye 61

5. Oral antibiotics Fluoroquinolones penetrate the eye well and moxifloxacin 400 mg daily
for 10 days is recommended;
6. Oral Steroids The rationale for the use of steroids is to limit destructive complications of
the inflammatory process.
7. Periocular antibiotic and steroids Dexamethasone or triamcinolone should be
considered if systemic therapy is contraindicated.
8. Topical Dexamethasone for anterior uveitis.
9. Topical Mydriatic such as atropine.
10. Intravitreal steroids may reduce inflammation in the short term but do not influence the
final visual outcome
11. Pars Plana Vitrectomy

Q-10. A forty years Old man presents with a dense vitreous hemorrhage after hammering nail.
Eye examination shows an entry wound of a foreign body. How will you manage such a case?

ANS:
Management:
1. Confirm the diagnosis by Slit Lamp Examination, B-scan
2. Evaluate for bleeding disorder
3. Conservative treatment:
ii) Antibiotics and NSAIDS
iii) Dressing and Care of wound
4. Surgery:
i) Vitrectomy, through pars plana if vitreous does not clear within three months. Vitreous
substitutes are, Air, physiological solution (Ringolactate), Expanding gases, Silicone oil

Q-11. A 40 years old female presents with gradual bulging of her right eye for 6 months. On ocular
examination she has axial proposes.
a) What is the differential diagnosis?
b) How will you investigate such a case?
c) How will you treat such a case?

ANS:
a) DIFFERENTIAL DIAGNOSIS:
1. Thyroid Ophthalmopathy due to Hyperthyroidism
2. Retro-orbital tumors
3. Retinoblastoma
4. Lymphoma
5. Leukemia
6. Raised intracranial pressure
b) INVESTIGATION:
 Laboratory investigation
1. Blood Cp and ESR
2. TSH T3 T4
3. ANCA
4. ACE

KMU Past Papers Solved SEQs


Orbit and Injury to Eye 62

 Imaging techniques
1. CT scan
2. MRI
3. X ray
4. Ultrasonography
5. Venography
6. Arteriography
 Histopathology
1. Fine needle aspiration
2. Incisional biopsy
3. Excisional biopsy
c) TREATMENT:
1. Treat Hyperthyroidism with Anti-thyroid drugs/ Radio-iodine/ Thyroidectomy
2. Treat Thyroid Ophthalmopathy with steroids, surgical decompression, Lid lengthening
surgery, Blepharoplasty
3. Chemotherapy for malignancy
4. Enucleation in severe cases

Q-12. A 10 years old male child presented with pain and loss of vision in right eye after getting
blunt trauma with a cricket ball a few minutes ago. There is sub-conjunctival hemorrhage in
the temporal quadrant, the visual acuity is hand movements and the pupil is not visible in eye.
a) What is your most probable diagnosis?
b) What is the most important parameter that you should watch it very closely and kept it
controlled?
c) If the above parameter is not kept controlled, what two serious complications can arise?

ANS:
a) HYPHEMA
b) INTRAOCULAR PRESSURE
c) COMPLICATIONS:
1. Glaucoma
2. Retinal Detachment

Q-13. A 40 years old woman complains of a sudden onset of double vision after being involved in an
accident two days ago. Examination revealed normal visual acuity and limitation of abduction side.
a) What is your diagnosis?
b) What is your differential diagnosis?
c) How will you treat her?

ANS:
a) ORBIT TRAUMA WITH MEDIAL RECTUS MUSCLE ENTRAPMENT
b) DIFFERENTIAL DIAGNOSIS:
1. Strabismus
2. Sixth nerve palsy
3. Basal skull fracture
4. Raised Intracranial Pressure (downward displacement of the brainstem, may stretch one or
both sixth nerves)

KMU Past Papers Solved SEQs


Ocular
Orbit and
mobility
Injuryand
to Eye
Strabismus 63

5. Meningitis
c) TREATMENT:
1. Observation with monocular occlusion or prismatic correction of diplopia is appropriate in
idiopathic and presumed microvascular lesions; up to 90% will recover spontaneously,
usually over weeks to several months.
2. Botulinum toxin injection into the ipsilateral medial rectus may be used to prevent
contracture, assess residual function and sometimes to facilitate prismatic correction with a
large deviation; it is rarely curative.
3. Surgery should be considered only when adequate time has been allowed for maximal
spontaneous improvement, typically at least 6–12 months from onset.
i) Partial palsy (paresis) is treated by adjustable medial rectus recession and lateral
rectus resection in the affected eye to maximize the field of binocular single vision.
ii) Complete palsy is treated by transposition of the superior and inferior recti to
positions above and below the affected lateral rectus muscle, coupled with
weakening of the ipsilateral medial rectus (sometimes with injection of botulinum
toxin – ‘toxin transposition’). Three rectus muscles should not be detached from the
globe at the same procedure because of the risk of anterior segment ischemia.
4. Permanent prism. Troublesome but mild residual deviation may be treated with a prism
incorporated into spectacles as an alternative to surgery.

Q-14. A young man presents in emergency room with ammonium hydroxide thrown in his face
half hour ago.
a) What are the early clinical features?
b) What are the late complications?
c) List the emergency room treatment and subsequent treatment options.

ANS:
a) Clinical Features:
1. Congestion and Chemosis of Conjuntiva
2. Conjunctival Redness
3. Limbal ischemia
4. Corneal laceration
b) LATE COMPLICATIONS:
1. Symblepharon
2. Corneal Ulcer
c) EMERGENCY ROOM TREATMENT:
1. Wash Eye thoroughly with plenty of clean water
2. Neutralize Ammonium Hydroxide with weak acids, eg. Boric Acid or Milk
Subsequent treatment options:
1. If Corneal Erosion present, treatment for Corneal Ulcer should be given
2. If Cornea is not involved, steroid drops and ointments should be used (prevent
Symbelpharon formation and reduce congestion and chemosis)
3. Irrigation with bland lotion, normal saline 3% soda bicarbonate or clean water
4. Use of Dart glasses

KMU Past Papers Solved SEQs


Ocular mobility and Strabismus 64

Q-1. A 65 years old diabetic and hypertensive patient developed sudden ptosis in left eye. The
ptosis is severe and the upper lid margin covers the pupil, When the lid is elevated manually
the eye is deviated laterally.
a) What is your diagnosis?
b) What Other signs will you look for?
c) What is your plan of treatment for this condition of the left eye?

ANS:
a) THIRD NERVE PALSY
b) SIGNS TO LOOK FOR
1. Profound ptosis due to weakness of the levator muscle so that diplopia may not be
volunteered.
2. Abduction and depression in the primary position due to unopposed action of the lateral
rectus and superior oblique muscles. The intact superior oblique muscle also causes intorsion
of the eye at rest, which increases on attempted down gaze.
3. Normal abduction as the lateral rectus is intact
4. Limited adduction due to medial rectus weakness
5. Limited elevation due to weakness of the superior rectus and inferior oblique
6. Limited depression due to weakness of the inferior rectus
7. Dilated pupil and defective accommodation due to parasympathetic palsy.
c) TREATMENT PLAN:

 Observation:
1. Is usually appropriate in presumed micro vascular cases (hypertension and diabetes) the
majority will resolve over weeks or months.
2. Temporary (Fresnel stick-on) prisms may be useful if the angle of deviation is small.
3. Uniocular occlusion may be necessary to avoid diplopia if the ptosis component is partial or
recovering.
4. Botulinum toxin injection into the uninvolved lateral rectus muscle is sometimes used to
prevent its contracture when recovery time is prolonged.
 Surgical treatment:
For the ocular motility element and ptosis should be done only after spontaneous improvement has
ceased, not earlier than 6–12 months from onset.
1. Blepharoplasty
2. Muller Muscle Muscle Resection
3. Levator Resection
4. Levator Advancement

Q-2. A 32 years Old woman complains of intermittent double vision. Which occur more often
towards the end of the day. Her lids droop significantly by evening. On awakening she does
not have these problems
a) What is the diagnosis?
b) How will you support the diagnosis?
c) What are the treatment Options?

ANS:
a) MYASTHENIA GRAVIS
b) SUPPORT DIAGNOSIS BY:

KMU Past Papers Solved SEQs


Ocular mobility and Strabismus 65

1. Ice pack test: This tests for an improvement after an ice pack is placed on the ptotic eyelid for
2 minutes, as cold inhibits the breakdown of acetylcholine by acetylcholinesterase.
2. Antibody testing: Supports a diagnosis of MG and predicts the likelihood of thymoma.
i) Acetylcholine receptor (AChR) antibodies
ii) MuSK protein antibodies
iii) Striational antibodies.
3. Edrophonium (Tensilon) test: Edrophonium is a short-acting anticholinesterase that
confers a transient improvement of weakness in MG.
4. Electromyography: Shows characteristic features of fatigue.
5. Muscle biopsy: Reveals neuromuscular junction antibodies and characteristic electron
microscopy features, but is not commonly performed.
6. Thoracic imaging: (MR, CT, CT/PET) to detect thymoma, lung tumour or an intracranial
mass for ocular myasthenia.
7. Thyroid function testing should be performed as autoimmune thyroid disease can be
associated
c) TREATMENT OPTIONS:
1. Medical:
1. Anticholinesterases (pyridostigmine)
2. Steroids
3. Immunosuppressant (Azathioprine)
4. Plasmapheresis
5. I/V Immunoglobulins
2. Surgical:
1. Thymectomy (if a thymoma is present)

Q-3. A seven years old child present with lazy eye


a) What are the possible causes of it?
b) How will you evaluate such case?

ANS:

a) POSSIBLE CAUSES:
1. Unilateral squint, resulting in continued monocular suppression of deviating eye
2. Stimulation deprivation. e.g. complete ptosis, corneal opacity, congenital cataract
3. Un-corrected Refractive error
4. Un-corrected astigmatism more than 1 diopter
b) EVALUATION:
1. History regarding any event/incident relating to cause of acquired squint
2. Visual acuity of both eyes
3. Pin Hole Test
4. Ophthalmoscopy
5. Slit Lamp Examination

KMU Past Papers Solved SEQs


Ocular mobility and Strabismus 66

Q-4. A 65-year-Old gentleman presents with Double vision. If he closes either eye the double
vision is relived, for the last 7 days he also had Left sided weakness of both upper and lower
limbs which was confirmed on neurological examination. He also complains of low grade
fever. On examination his Visual acuities are 6/18 both eyes improving to 6/9 partial with pin
hole. On Hirschberg testing he has Esotropia of 10 degrees. The Right eye has very limited
abduction but other movements are normal Ocular movements of the Left Eye are normal
there is pupillary deficit and NO Papilloederna.
a) What cranial nerve is affected in the Right Eye?
b) On the basis of ocular and neurological signs which part of the brain is likely to be affected?
c) Enumerate Diseases that cause cranial Nerve palsies affecting Ocular Movements.
d) Enumerate investigations you would carry out in this case and the rationale for carrying out
that particular investigation (in this particular case)?

ANS:
a) ABDUCENT NERVE
b) PONTO-MEDULLARY JUNCTION AT BRAIN STEM
c) CAUSES OF CRANIAL NERVE PALSIES
1. Raised Intracranial Pressure (downward displacement of the brainstem, may stretch one or
both sixth nerves)
2. Acoustic neuroma
3. Basal skull fracture
4. Nasopharyngeal` tumor
5. Gradeingo syndrome
6. Meningitis
7. Syphilis
8. Aneurysms in Circle of Willis
d) INVESTIGATIONS:
 Hematological test:
1. Total and differential white blood cell count (TLC and DLC) is done to have a general
information about inflammatory response of the body.
2. ESR for the existence of any chronic inflammatory condition.
3. Blood sugar levels to rule out diabetes mellitus.
4. Serology for syphilis
Lumbar puncture (LP) for meningitis and to find out intracranial pressure.
 Imaging:
1. CT- scan for basal skull fracture and nasopharyngeal tumor.
2. MRI to exclude a space-occupying lesion, enlarged ventricles and herniation of the
brainstem.
3. MRA for aneurysms in the intra-cranial arteries.

Q-5. A 4 years old child is brought by parents with a convergent squint in the right eye. On
examination visual acuity is 6/36 (or defective) in both eyes.
a) What is the differential diagnosis?
b) How will you evaluate such a case?

ANS:
a) DIFFERENTIAL DIAGNOSIS:
1. Amblyopia,

KMU Past Papers Solved SEQs


Ocular mobility and Strabismus 67

2. congenital cataract,
3. Astigmatism
4. Hypermetropia,
5. Convergence center hyper-activity,
6. Paralysis of right Abducent nerve
b) MANAGEMENT:
1. History and Examination,
2. Investigations
3. Retinoscopy,
4. FBC with ESR,
5. CT-scan
6. MRI of brain

Q-6. A 10 years old child is brought to you with trauma to his right eye with a knife. On examination
of the anterior segment with torch light, you see a small blackish mass protruding at the nasal
limbus.
a) What is your diagnosis?
b) What immediate measures will you take?
c) What is the ultimate management?
d) Name single most vision threatening complication if management is delayed?

ANS:
a) UVEAL PROLAPSE
b) IMMEDIATE MEASURES:
1. An eye shield should be applied to prevent further damage.
2. Medical treatment is only indicated when the prolapse is small, is covered by the
conjunctiva
3. Intravenous antibiotics should be considered because infection from an iris
prolapse can spread to the intraocular contents.
4. Tetanus toxoid may be considered depending on the immunization status and the
wound type.
c) MANAGEMENT
1. Small, self-sealing, penetrating wounds are best managed with topical antibiotics
and patching for a short time.
2. Larger wounds might need tissue glue, but most will require suturing.
3. watertight closure of the wound, and cause as little distortion as possible of the
normal corneal dome shape.
4. Place full-thickness, interrupted, 10‑0 nylon sutures
5. Long, tighter sutures in the periphery of the cornea and shorter, looser sutures
towards the center tend to preserve a more physiologic corneal shape.
6. Make sure the wound is free of all foreign material and tissue.
7. Replace an extruded iris into the anterior chamber unless it is obviously necrotic
or contaminated
8. Once the eye is hermetically sealed, restore the IOP by injecting balanced salt
solution, preferably through an area where the needle-tip can be visualized,
9. Give topical and systemic broad spectrum antibiotics
10. Follow up is necessary
Note: General anesthesia should be used during surgery.
d) SINGLE MOST COMPLICATION

KMU Past Papers Solved SEQs


Ocular mobility and Strabismus 68

Endophthalmitis

Q-7. A 30 years old male presented with acquired immune deficiency syndrome
a) What are the modes of transmission?
b) What are its ocular manifestations?

ANS:

a) MODE OF TRANSMISSION:
2. Sexual contact with an infected person
3. Sharing needles syringes or other injection equipment with someone who is infected
4. Mother to child transmission
5. Transmission in health care settings
6. Transmission via donated blood or blood clotting factors however this is now very rate in
countries where blood is screened for HIV antibodies
b) OCULAR MANIFESTATION:
A less CD4+ T-Cell count is associated with the following
1. Retinal or conjunctival microangiopathy
2. Anterior uveitis
3. Toxoplasmic retinochoroiditis
4. Pneumocystis choroiditis,
5. Histoplasma chorioretinitis,
6. Cryptococcal choroiditis,
7. Mycobacterium avium complex infection
8. Cryptococcosis
9. Microsporidoiosis
10. HIV encephalopathy
11. Progressive multifocal leukoencephalopathy

Q-7. A 2 years old child is presented with right convergent squint of recent onset.
a) What are the probable causes?
b) HOW will you manage it in relation to the most common cause?

ANS:
a) CAUSES:
1. Congenital and Acquired
2. Non- paralytic:
i) Refractive errors
ii) Hypermetropia
iii) Astigmatism
iv) Convergence center hyperactivity
3. Paralytic: paralysis of the right abducent nerve
b) MANAGEMENT:
7. Eye Patching
1. Eye Glass or Contacts lenses
2. Prisms
3. Vision Therapy
4. Surgery

KMU Past Papers Solved SEQs

You might also like